Term 4 DERS

¡Supera tus tareas y exámenes ahora con Quizwiz!

A 22-year-old man comes to the physician because of severe lower right quadrant pain, malaise, diarrhea and weight loss. An endoscopy shows disease in the terminal ileum which is patchy with normal intervening mucosa. His physician would like to start him on an aminosalicylate but the patient expresses concern about the side effects. The physician reassures him that this one is well tolerated. Which of the following drugs is most likely prescribed for the patient? A. Mesalamine B. Budesonide C. Mercaptopurine D. Sulfasalazine E. Methotrexate

A. Mesalamine

An 18-year-woman comes to the gynecologist because of a 5-week history of amenorrhea. She attained menarche at 15 years of age and has had regular menstrual cycles in the past. A urine pregnancy test is positive. Laboratory studies shows abnormally elevated β HCG levels in the serum. Transvaginal ultrasonogram shows a snowstorm appearance in the uterine cavity without any identifiable fetal parts. Which of the following will most likely phenotype in this condition? A. 46XY B. 69 XXY C. 69 XXX D. 69 XYY E. 47 XXY

A. 46XY

2. A 54- year-old woman comes to the physician because of a 2-week history of abdominal distinction and pain on the right side. She also has had a 7-kg (15.4-lb) weight loss in the past 6 months. Physical examination shows a palpable large cystic mass in the right iliac fossa and diffuse abdominal tenderness. Laparotomy shows extensive mucinous ascites due to a ruptured mucinous cystadenocarcinoma of the right ovary. Which of the following additional tumors can most likely cause this complication? A. A. Appendicular carcinoma B. B. Serous cystadenocarcinoma of ovary C. C. Diffuse gastric adenocarcinoma D. D. Benign serous cystadenoma E. E. Endometroid endometrial carcinoma

A. A. Appendicular carcinoma

3. A 62-year-old woman comes to the physician because of passage of blood-tinged vaginal discharge. Her last menstrual period was 10 years ago. She has a 20-year history of type 2 diabetes mellitus. Her temperature is 37°C (98.6°F), pulse is 86/min, respirations are 18/min and blood pressure is 130/86 mm Hg. Bimanual pelvic examination shows asymmetric enlargement of uterus and no palpable adnexal masses. There are no cervical erosions or masses. Endometrial biopsy shows a back-to-back arrangement of large pleomorphic glands lined with dysplastic epithelium and surrounded by very little intervening stroma. Which of the following is the most likely pre-disposing factor for the development of this patient's condition? A. A. Estrogen replacement therapy B. B. Human papilloma virus (HPV) infection C. C. Multiple sexual partners D. D. Cigarette smoking E. E. Alcohol abuse

A. A. Estrogen replacement therapy

1. A 55-year-old woman comes to the physician because of a 1-month history of painless lumps in both breasts. She has had a 5-kg (11-lb) of weight loss over the past 2 months. Physical examination shows vaguely palpable, non-tender, fixed masses in both the breasts and axillary lymphadenopathy. Histologic examination of a biopsy from the mass shows signet-ring tumor cells arranged in a single file pattern. The tumor cells surround the ducts and acini in a bulls-eye pattern. Which of the following genetic alternations is most likely associated with this cancer? A. A. Loss of E-cadherin B. B. PTEN mutations C. C. BRCA1 mutation D. D. Over expression of c-myc E. E. Translocation t(8:14)

A. A. Loss of E-cadherin

*A 34-year-old man comes to the emergency department because of a 2-day history of fever, abdominal cramps, and bloody diarrhea. His stool was watery for the first day of his illness. Travel history is negative. His temperature is 38.3oC (100.9oF), pulse is 90/min, respirations are 15/min, and blood pressure is 115/75 mm Hg. Physical examination shows generalized tenderness without guarding. Laboratory studies of his stool show Gram negative, non-motile rods, that is H2S negative. Which of the following microbial characteristics best describes the causal agent? A. Acid stable B. Halotolerant C. Aerobic D. Microaerophilic E. Psychrophilic

A. Acid stable

2. A 49-year-old man is brought to the emergency department because of a 6-hour history of severe upper abdominal pain, nausea and vomiting. The pain is radiating to the back. He is a chronic alcoholic and smokes occasionally. Physical examination shows an ill-looking middle-aged man in severe distress with marked epigastric tenderness. Laboratory studies show markedly elevated serum amylase levels. Which of the following is the most likely complication of this condition? A. Acute respiratory distress syndrome B. Carcinoid syndrome C. Perforation of the stomach D. Migratory thrombophlebitis E. Gallstones

A. Acute respiratory distress syndrome

A 33-year-old man comes to the physician because of a 3-month history of upper abdominal burning and pain. The pain is worse at night and is relieved by eating. He has tried antacids and H2 receptor blockers without complete resolution of symptoms. Vital signs are within normal limits. Physical examination shows mild epigastric tenderness to deep palpation. An upper GI endoscopy shows a well demarcated break in the mucosa of the first portion of the duodenum. Fecal antigen testing shows an active infection. Which of the following drugs would most likely prevent recurrence of the duodenal lesion? A. Amoxicillin B. Hydrocortisone C. Famotidine D. Omeprazole E. Metoclopramide

A. Amoxicillin

A 15-year-old girl comes to the physician for a follow-up. She has a history of type 1 diabetes mellitus. Before consuming a meal, she estimates the meal's approximate carbohydrate load, and administers the appropriate insulin to control postprandial glucose levels. Which of the following types of insulin would be most appropriate to accomplish this goal? A. Aspart B. Glargine C. NPH D. Detemir E. Regular

A. Aspart -consume before a meal and short onset

6. A 40-year-old woman is brought to the emergency department because of a 4-day history of pain in the upper abdomen and yellowish discoloration of the skin. Physical examination shows scleral icterus and tenderness in the right upper quadrant of the abdomen. Laboratory studies show elevated serum levels of liver enzymes and presence of serum anti-nuclear antibodies. Ultrasonography of the abdomen shows hepatomegaly. Which of the following etiopathogenetic mechanisms is most likely associated with the development of this condition? A. Autoimmune destruction of the hepatocytes B. Direct damage due to viral infection C. Accumulation of a mutant protein within the hepatocytes D. Damage to the hepatocytes due to drug toxicity E. Use of contraceptive pills

A. Autoimmune destruction of the hepatocytes

10. A 40-year-old woman is brought to the emergency department because of a 4-day history of pain in the upper abdomen and yellowish discoloration of the skin. Physical examination shows scleral icterus and tenderness in the right upper quadrant of the abdomen. Laboratory studies show elevated serum levels of liver enzymes and presence of serum anti-nuclear antibodies. Ultrasonography of the abdomen shows hepatomegaly. Which of the following etiopathogenetic mechanisms is most likely associated with the development of this condition? A. Autoimmune destruction of the hepatocytes B. Direct damage due to viral infection C. Accumulation of a mutant protein within then hepatocytes D. Damage to the hepatocytes due to drug toxicity E. Use of contraceptive pills

A. Autoimmune destruction of the hepatocytes - Autoimmune hepatitis

8. A 45-year-old man comes to the physician because of a 3-week history of progressive abdominal pain, bloating and decreased appetite. He describes the pain as crampy and localized the upper abdomen. It often improves immediately after meals or taking antacids. He has occasional nausea but denies fever, vomiting or diarrhea. Temperature is 37oC (98.6oF), pulse is 84/min, respirations are 14/min and blood pressure is 110/70mmHg. Physical examination shows mild epigastric tenderness with no rebounding, and normal bowel sounds. The physician suspects a diagnosis of peptic ulcer disease caused by a bacterial infection. Which of the following virulence factors is associated with attachment of the most likely causal agent? A. BabA B. Toxin A C. VacA D. Urease E. Toxins A & B

A. BabA

A 25-year-old man comes to the physician because of a 6-week history of lower abdominal pain and intermittent bloody stools. He has not had any sick contacts, no weight loss and no history of foreign travel. His vital signs are within normal limits. A colonoscopy shows rectal and sigmoidal pseudopolyps with erythematous, friable mucosa. Which of the following is the most appropriate initial treatment for this patient? A. Balsalazide B. Infliximab C. Natalizumab D. Loperamide E. Azathioprine

A. Balsalazide

A 70-year-old man comes to the physician because of a 4-week history of worsening bloating, nausea, heartburn, weight loss and a dull pain in his stomach. The pain is worse about 5 hours after eating and sometimes wakes him up at night. He has smoked 1 pack of cigarettes per day for the last 50 years. A non-invasive test is positive. Which of the following exhaled C13 -labelled compounds is most likely being detected by this test? A. CO2 B. Urease C. Ammonia D. Urea E. CagA

A. CO2

8. A 36-year-old woman comes to the physician because of a 2-month history of progressive shortness of breath and easy fatigability. Her maternal aunt died of colon cancer 5 years ago. Physical examination shows conjunctival pallor. Colonoscopy shows a hemorrhagic fungating mass in the ascending colon. A biopsy is taken from the mass and molecular analysis of the cells shows an MSH2 gene mutation with microsatellite instability. This patient is most likely at risk of developing which of the following conditions? A. Endometrial carcinoma B. Liver cirrhosis C. Primary sclerosing cholangitis D. Hepatic adenoma E. Pancreatic insufficiency

A. Endometrial carcinoma

*A 22-year-old man comes to the emergency department because of a 2-day history of severe abdominal pain and bloody mucoid diarrhea. He states that he ate a "pink" chicken burger from a local restaurant 6 days ago. His temperature is 37.3oC (99.1oF), pulse is 87 /min, respirations are 15/min, and blood pressure is 115/75mmHg. Abdominal examination shows generalized tenderness with no rigidity or rebound tenderness. Laboratory studies show numerous leukocytes, RBCs and Gram negative, motile, curved, microaerophilic bacilli in the stool. Which of the following etiological agents is most likely responsible for his symptoms? A. Campylobacter jejuni B. Shiga-toxin producing Escherichia coli C. Clostridium perfringens D. Vibrio parahemolyticus E. Shigella sonnei

A. Campylobacter jejuni

A 24-year-old man comes to the physician because of a 3-day history of painful urination and a penile discharge. He is living with his new partner who he met 3-weeks ago. His vitals are temperature 37.2°C (99°F), respirations 16/min, pulse 80/min and blood pressure 125/75mmHg. Physical exam shows a mucoid penile discharge and unilateral testicular swelling. Laboratory studies on a sample of the discharge show a Gram negative obligate intracellular organism, with no growth on Modified Thayer-Martin agar. Which of the following is the most likely diagnosis in this patient? A. Chlamydia B. Ureaplasma C. Gonorrhea D. Chancroid E. Genital herpes

A. Chlamydia

3. A 32-year-old woman comes to the physician because of a 2-week history of colicky pain in the right hypochondrium. Physical examination shows scleral icterus. Laboratory studies show increased levels of serum conjugated bilirubin. Imaging studies show distended common bile duct. Which of the following is the most likely complication for this condition? A. Cholangitis B. Pancreatic insufficiency C. Hepatocellular carcinoma D. Hepatic adenoma E. Chronic pancreatitis

A. Cholangitis

7. A 25-year-old man comes to the physician because of a 4-month history of repeated episodes of pain in the abdomen. Physical examination shows hyperpigmented lesions on his lips, gums and perioral areas. Imaging studies show several polyps in the small and large intestine. A biopsy from one of the polyps show arborizing network of smooth muscles between glands that are lined by non-dysplastic epithelium. This patient is at risk of developing which of the following most likely complications? A. Colonic adenocarcinoma B. Liver cirrhosis C. Primary sclerosing cholangitis D. T cell lymphoma E. Diverticulosis

A. Colonic adenocarcinoma -> Peutz Jehger Polyps

A 64-year-old man comes to the physician because of a 4-week history of pain on swallowing. He has recently received a kidney transplant and is on immunosuppressive drugs. Physical examination of the oro-pharynx is unremarkable. Endoscopic investigations show a single, circular esophageal lesion of 2.5cm diameter (see arrow on image) at distal portion of his esophagus. Which of the following is the most likely causative agent? A. Cytomegalovirus B. Viridans Streptococcus C. Candida spp. D. Herpes Simplex Virus I E. Helicobacter pylori

A. Cytomegalovirus

A 43-year-old man comes to the physician because of a 3-month history of loss of energy, fatigue, cold intolerance, and weight gain. Physical examination shows cold, dry, scaly skin, peripheral edema of hands and feet, and a puffy face. The thyroid gland is firm and enlarged. Which of the following signs or symptoms are also most likely to be found in this patient? A. Delayed tendon reflex relaxation B. Tachycardia and irritability C. Polyuria and diarrhea D. Tremor, goiter, and moist skin E. Hyperactivity and palpitations

A. Delayed tendon reflex relaxation

1. A 50-year-old woman comes to the physician because of a 2-month history of itching in her right nipple. She was diagnosed with diabetes mellitus 4 years back. Physical examination shows an erythematous eruption with a scaly crust on the right nipple and areola. Laboratory studies show no abnormalities. A biopsy from the affected area shows cells with large hyperchromatic nucleus and halo in the epidermis. Which of the following is the most likely underlying condition associated with this disease? A. Ductal carcinoma in situ B. Lobular carcinoma C. Cystosarcoma phyllodes D. Fibroadenoma E. Intraductal papilloma

A. Ductal carcinoma in situ

An 11-year-old boy is brought to the physician by his father because of a 2-week history of loss of appetite, abdominal pain and vomiting. The father also noticed his son develop a swollen abdomen for the past 6 months. The family lives in a small village in eastern Congo, Africa. His vital signs are within normal limits. Physical examination shows a distended, soft but mildly tender abdomen and hyperactive bowel sounds. Laboratory studies show anemia and mild leukocytosis. An exploratory laparotomy shows a dilated small bowel obstructed by bundles of live, long, slender, creamish-white worms measuring 20-30 cm long. In the life cycle of this pathogen, which of the following is most likely the infective form? A. Egg B. Cyst C. Trophozoite D. Microfilaria E. Sporozoite

A. Egg

10. A 28-year-old woman comes to the physician because of a 3-day history of generalized rash including her palms. She also described a small, painless ulcer like lesion on her genitalia that resolved 3-weeks ago. She is currently sexually active with her new partner and admits to inconsistent condom use. Her temperature is 37.2°C (98.6°F), respirations are 16/min, pulse is 75/min and blood pressure is 129/75 mm Hg. Physical examination shows a generalized coppery-red, maculopapular rash which includes her palms and soles. Which of the following microbial characteristics best describes the causal agent? A. Endoflagellated B. Pleomorphic C. Obligate aerobe D. Halophilic E. Gram variable

A. Endoflagellated

5. A 36-year-old woman comes to the physician because of a 2-month history of progressive shortness of breath and easy fatigability. Her maternal aunt died of colon cancer 5 years ago. Physical examination shows conjunctival pallor. Colonoscopy shows a hemorrhagic fungating mass in the ascending colon. A biopsy is taken from the mass and molecular analysis of the cells shows an MSH2 gene mutation with microsatellite instability. This patient is most likely at risk of developing which of the following conditions? A. Endometrial carcinoma B. Liver cirrhosis C. Primary sclerosing cholangitis D. Hepatic adenoma E. Pancreatic insufficiency

A. Endometrial carcinoma

A 26-year-old woman visits the gynaecologist because of amenorrhea for a period of 2 years. Her last menstrual cycle was 5 months ago. Vital signs are within normal limits. Physical examination shows acanthosis over the armpits and posterior neck, hirsutism and a BMI of 30. Laboratory studies show elevated serum estrogen and androgens. A pelvic ultrasound shows multiple cysts in right and left adnexa. Which of the following is a likely complication of her condition? A. Endometrial hyperplasia B. Hypoglycemia C. Luteal cysts D. Ovarian epithelial tumour E. Germ line tumour

A. Endometrial hyperplasia

7. A 60-year-old man comes to the physician because of a 1-week history of worsening pain on swallowing and upper abdominal discomfort. He denies nausea or vomiting. He was diagnosed 10 years ago with HIV, and has been receiving long-term treatment. His vital signs are all within normal limits. Physical examination shows slightly dry mucosal membranes, and there is no edema or erythema observed in the oropharynx. Endoscopy shows multiple small vesicles and ulcers less than 2cm in distal esophagous. Which of the following features characterizes the most likely causal agent? A. Enveloped double-stranded DNA genome B. Budding yeast and pseudohyphae C. Gram negative bacilli D. Mould with septate hyphae and frilled conidiophores E. Thick-walled oocysts

A. Enveloped double-stranded DNA genome = HSV-1 2.5 cm = CMV

A 43-year-old comes to the physician because of a 3-week history of fever, abdominal pain, nausea and vomiting. He was diagnosed with HIV 2-years ago and is on antiretroviral therapy. His temperature is 39°C (102.2°F), pulse is 130/min, respirations are 18/min and blood pressure is 110/76 mmHg. Physical examination shows scleral icterus and right upper quadrant tenderness. Laboratory studies show a CD4 count of 450 cells/mm3 and elevated levels of bilirubin, liver enzymes and prothrombin time. A qRT-PCR analysis shows 5.0 × 105 HIV RNA copies/mL and 2.0 × 105 HCV RNA copies/ml. Which of the following best describes the genome of the newly acquired infectious agent? A. Enveloped, non-segmented, single stranded, positive sense RNA B. Non-enveloped, positive sense RNA C. Non-enveloped, segmented, double stranded RNA D. Enveloped, segmented, single stranded, negative sense RNA E. Enveloped, non-segmented, single stranded, negative sense RNA

A. Enveloped, non-segmented, single stranded, positive sense RNA

A 52-year-old obese man comes to the physician because of frequent episodes of hypoglycemia. He has a history of type 2 diabetes mellitus and current medications are glyburide and metformin. Glyburide was discontinued and an agent that is less likely to produce hypoglycemia, and more likely to produce weight loss is started. Which of the following medications was started in this patient? A. GLP-1 agonist B. Sulfonylurea C. Thiazolidinedione D. Meglitinide E. Insulin

A. GLP-1 agonist B, C and E cause weight gain

11. A 2-year-old boy is brought to the physician by the mother for routine check-up. His pulse is 90/min, blood pressure is 100/80 mm Hg, and respirations are 24/min. Physical examination shows absence of the right testis in the scrotal sac. A pelvic ultrasound shows the presence of the right testes in the inguinal region. Which of the following complication is most likely associated with this condition, if left untreated? A. Germ cell tumor B. Sertoli cell tumor C. Lymphoma D. Leydig cell tumor E. Carcinoma of the penis

A. Germ cell tumor

4. A 50-year-old man comes to the physician because of a 2-month history of progressive bone and joint pains. He has noticed enlargement of his hands and feet during the past 4 months. His temperature is 36.7°C (98°F), pulse is 70/min, respirations are 14/min and blood pressure is 130/85 mm Hg. Laboratory studies show elevated serum levels of insulin-like growth factor 1 (IGF-1). The serum levels of the growth hormone is not suppressed during an oral glucose tolerance test. Which of the following additional clinical findings is most likely in this patient? A. Hyperglycemia B. Gigantism C. Exophthalmos D. Kidney stones E. Myxedema

A. Hyperglycemia

7. A 24-year-old woman comes to the physician because of a 2-day history of increased vaginal discharge. She denies any fever or chills. She is in a monogamous relationship with her husband for the past 5-years. Her temperature is 37.2°C (99.0°F), respirations are 16/min, pulse is 80/min and blood pressure is 121/75 mm Hg. Physical examination shows a thin, grey malodorous discharge on the vaginal wall. Laboratory studies of the discharge shows vaginal epithelial cells covered with bacteria. Which of the following best describes the pathogenesis of this infection? A. Increased Vaginal pH. B. Antigenic variation of type IV pili. C. Attachment via agglutinin-like protein. D. Elementary body cellular uptake. E. Latency in Sacral ganglion

A. Increased Vaginal pH.

3. A 60-year-old man comes to the physician because of a 3-month history of difficulty in swallowing. His symptom is associated with an unintentional weight loss of 12-lb (5.4-kg) over the same duration. An endoscopy of the esophagus shows presence of an exophytic mass in the lower end. Biopsy taken from the mass shows neoplastic gland-forming cells invading the basement membrane. Which of the following conditions is associated with this disease? A. Intestinal metaplasia B. Absence of ganglions C. Intraepithelial neutrophils D. Presence of eosinophils E. Presence of esophageal webs

A. Intestinal metaplasia

A 35-year-old man comes to the ED because of a 3-day history of crampy abdominal pain and bloody diarrhea. He said he returned 5 days ago from a 2-week trip to Sri-Lanka (SE Asia) and that the diarrhea began as watery and has had the presence of blood since last night. His T 38.8oC (101.80F), PR 118/min, RR 16/min BP 108/62mmHg. Laboratory examination of the stools shows the presence of erythrocytes and leukocytes and routine stool culture revealed the causative bacterium. Which of the following pathogenic mechanisms is most likely responsible for his symptoms? A. Invasion by toxin producing bacteria B. SI colonization and effacement C. Vagus nerve stimulation from pre-formed toxin. D. Enterotoxin production. E. Intestinal secretion without mucosal disruption

A. Invasion by toxin producing bacteria

A 46-year-old woman comes to the physician because he has been feeling tired and lethargic for the past four months. She also mentions that she is always turning the heat up at home while her husband is sitting around in a t-shirt. She takes no medications. Physical examination shows dry skin, periorbital edema and a normal thyroid gland. Her TSH is 10.1 (normal 0.5-5.0 µU/mL). She is diagnosed with hypothyroidism. Which of the following is the most appropriate pharmacotherapy for this patient? A. Levothyroxine B. Triiodothyronine C. Methimazole D. Propylthiouracil E. Potassium iodide

A. Levothyroxine

A 35-year-old man comes to the physician because of a 3-day history of fever, watery diarrhea, and abdominal pain. He works on a dairy farm and have a history of occasional consumption of unpasteurized milk. His Temperature is 38.5°C (101.3 °F), pulse is 90/min, respirations are 15/min, and blood pressure is 121/80mmHg. Physical examination shows generalized abdominal tenderness without guarding. Stool analysis shows the presence of fecal leucocytes and the presence of a Gram-positive rod, with tumbling motility. Which of the following is most likely causal agent? A. Listeria monocytogenes B. Escherichia coli C. Shigella sonnei. D. Vibrio cholerae. E. Bacillus cereus

A. Listeria monocytogenes

A 42-year-old man comes to the physician because of a 6-week history of increasing fatigue, loss of appetite and frequent urination. He also has a 2-week history of swelling in the feet and face. He has a 9-year history of uncontrolled type 2 diabetes mellitus. His temperature is 37⁰C (98⁰F), pulse is 78/min, respirations are 17/min and blood pressure is 142/82 mmHg. Physical examination shows facial swelling and pedal edema. Laboratory studies show Serum creatinine:2.3mg/dL (n: 0.8-1.2mg/dL), Fasting plasma glucose:210 mg/dL and hemoglobin A1c: 8%. Which of the following antidiabetic medications will most likely be contraindicated in this patient? A. Metformin B. Glyburide C. Acarbose D. Exenatide E. Repaglinide

A. Metformin

***A 56-year-old man comes to the physician because of a 3-day history of dyspepsia and epigastric pain. He recently took more than the recommended dosage of NSAIDs because of pain associated with a chronic degenerative condition of the hip and knee joints. Physical examination shows moderate epigastric tenderness. Endoscopic biopsy specimen from the gastric mucosa is obtained. Which of the following histological features is most likely to be seen in this patient's condition? A. Mucosal erosions, hyperemia, punctate areas of hemorrhage and inflammatory infiltrates B. Deep mucosal ulcer with fibrosis and collagenous scaring C. Neoplastic cells forming glands invading stroma. D. Sheets of mucin-filled cells with signet ring appearance E. Whorls and bundles of spindle-shaped or epithelioid cells

A. Mucosal erosions, hyperemia, punctate areas of hemorrhage and inflammatory infiltrates

A 56-year-old man comes to the physician because of a 3-day history of dyspepsia and epigastric pain. He recently took more than the recommended dosage of NSAIDs because of pain associated with a chronic degenerative condition of the hip and knee joints. Physical examination shows moderate epigastric tenderness. Endoscopic biopsy specimen from the gastric mucosa is obtained. Which of the following histological features is most likely to be seen in this patient's condition? A. Mucosal erosions, hyperemia, punctate areas of hemorrhage and inflammatory infiltrates B. Deep mucosal erosions with fibrosis and collagenous scaring C. Pleomorphic cells forming "back-to-back glands" D. Sheets of mucin-filled cells with signet ring appearance E. Whorls and bundles of spindle-shaped or epithelioid cells

A. Mucosal erosions, hyperemia, punctate areas of hemorrhage and inflammatory infiltrates

1. A 23-year-old woman comes to the physician because of a 2-month history of amenorrhea. She is married and had been trying to conceive for the past 8 years. She attained menarche at the age of 13 years and has a history of irregular menstruation for the past 1-year. Physical examination shows an obese woman with coarse facial hair. A urine pregnancy test is negative. Ultrasonography of the abdomen shows bilaterally enlarged ovaries with sub-capsular cysts. Which of the following best explains the pathogenesis of this condition? A. Persistently elevated LH/FSH ratio B. Excess androgen and low estrogen levels C. Low androgen and excess estrogen levels D. Excess secretion of prolactin E. Excess progesterone secretion

A. Persistently elevated LH/FSH ratio

6. A 60-year-old woman comes to the physician because of a 2-week history of increasing muscle weakness and fatigue. Her temperature is 37°C (98.6°F), pulse is 100/min, blood pressure is 160/100 mm Hg and respirations are 16/min. Her body mass index is 40 kg/m2. Physical examination shows purple abdominal striae, facial plethora, facial hair, moon face and a hump over the back of the neck. Laboratory studies show elevated serum cortisol and ACTH levels. A low-dose dexamethasone injection fails to decrease the serum cortisol level, but a high-dose dexamethasone injection decreases the serum cortisol levels by more than 50%. Which of the following is the most likely cause of this patient's condition? A. Pituitary adenoma B. Chronic alcoholism C. Adrenal adenoma D. Carcinoma of the lung E. Exogenous steroid use

A. Pituitary adenoma

A 27-year-old man comes to the physician because of a 5-day history of painless lesion on his penis. He is sexually active with his girlfriend that he met 2-months ago with inconsistent condom use. His temperature is 37.2°C (99°F), respirations are 16/min, pulse is 80/min and blood pressure is 121/75 mmHg. Physical examination shows a small, non-tender ulcer on the penile shaft. Which of the following is the most likely diagnosis? A. Primary syphilis B. Secondary syphilis C. Tertiary syphilis D. Early Latent syphilis E. Late Latent syphilis

A. Primary syphilis

A 7-month-old male infant is brought to the ED by his parents because of multiple episodes of watery diarrhea, fever and vomiting for the past 10 days. The infant was born full term (5.3 lbs), he attends day care is formula fed and immunized appropriately for his age. His HR 150/min, RR 34/min, he had a low volume pulse, BP is 80/50mmHg. He is ill looking, has sunken eyes and demonstrates a delayed skin pinch. Stool culture shows a Gram -ve, non-motile rods, that does not ferment lactose. Which of the following pathogens is most likely the causative agent of his symptoms? A. Shigella sonnei B. Enteroinvasive Escherichia Coli C. Salmonella Serotype Enteritidis D. Enteropathic Escherichia Coli E. Shiga toxin producing E. Coli

A. Shigella sonnei

1. A 28-year-old woman comes to the physician because of a painful lump in her left breast for the past 3 days. She denied any history of trauma. Physical examination shows a red, tender breast with cracks and fissure in the nipple. She has been breastfeeding her child from past 20 days. Which of the following best explains the etiology of this condition? A. Staphyococcus aureus B. H. Influenza C. S. Epidermidis D. E.Coli E. Histoplasma

A. Staphyococcus aureus

Q1. A 30-year-old woman comes to the physician because of a 1-week history of fever, abdominal pain and diarrhea. She has had recurrent episodes of similar symptoms in the past 5 years. Barium studies show intermittent areas of narrowing of the jejunum and ileum. An endoscopy shows a cobble-stone appearance of the mucosa with linear serpentine ulcers in the terminal ileum. Which of the following morphological features are most likely expected to be seen in a biopsy from the affected areas of the intestine? A. Transmural inflammation with noncaseating granulomas B. Villous atrophy with elongated crypts and intraepithelial lymphocytes C. Flask shaped ulcers containing trophozoites D. Distended macrophages with PAS positive granules in lamina propria E. Damaged crypts distended with exudate and pseudomembrane formation

A. Transmural inflammation with noncaseating granulomas

A 32-year-old woman is being evaluated for irregular menstrual periods for the past 5 months. She attained menarche at the age of 15 and used to have regular a 28-day cycle with 4 days of menses. Since the past 5-months, her cycle length has varied between 30-39 days and she has heavy menstrual bleeding for 5-7 days. She says that she has never been pregnant. Physical examination shows hirsutism and conjunctival pallor. Laboratory studies show an increased LH:FSH ratio. The abnormal uterine bleeding in this case can be attributed to which of the following? A. Unopposed estrogenic stimulation B. Granulosa cell tumor C. Uterine leiomyoma D. Ectopic pregnancy E. Coagulation disorders

A. Unopposed estrogenic stimulation

A 24-year-old man comes to the physician because of a 2-day history of fever and penile discharge. He lives with his new girlfriend and uses condoms inconsistently. His temperature is (38.2°C) 100.8°F, respirations are 16/min, pulse is 80/min and blood pressure is 121/75 mm Hg. Physical examination shows a purulent penile discharge. Laboratory studies of the discharge show intracellular Gram -ve diplococci. Which of the following factors most likely contributed to the cause of his symptoms? A. Unprotected sexual contact and transmission of an exogenous pathogen B. Imbalance of host microflora C. Overgrowth of an opportunistic pathogen D. Poor personal hygiene habits E. Overgrowth by endogenous microbial population

A. Unprotected sexual contact and transmission of an exogenous pathogen

A 28-year-old woman comes to the physician because of 3-day history of small itchy vesicles on her genitalia. She is sexually active with her new boyfriend and uses condoms occasionally. Her temperature is 37.2°C (99°F), respirations are 16/min, pulse is 80/min and blood pressure is 121/75 mm Hg. Physical examination shows multiple small vesicular lesions and erythematous papules on her external genitalia. Laboratory studies show an enveloped virus. Which of the following best describes the genome of the causal agent? A. ds DNA B. ss RNA C. ds RNA D. ss DNA E. ss circular DNA

A. ds DNA

A 28-year-old woman comes to the physician because of 1-week history of "small fleshy bumps" on her genitalia. She is sexually active with her new boyfriend and uses condoms occasionally. There is no history of HPV vaccination. Her temperature is 37.2°C (99°F), respirations are 16/min, pulse is 80/min and blood pressure is 121/75 mm Hg. Physical examination shows multiple small raised "cauliflower like" lesions on her external genitalia. Laboratory studies show a naked non-enveloped virus. Which of the following best describes the genome of the causal agent? A. ds-DNA B. ss-RNA C. ds-RNA D. ss-DNA E. ss circular DNA

A. ds-DNAk

6. A 65-year-old man comes to the physician because of a 5-month history of waking up multiple times at night with an urgent need to urinate but only passing small amounts of urine each time. He also admits to having to stand closer to the toilet bowl than before while he urinates. He has no other conditions. His vitals are within normal limits. Physical examination shows a nontender, non-nodular prostate gland measuring 3-4 fingerbreadths wide. After refusing surgical intervention, he is prescribed a drug that decreases conversion of testosterone to DHT. Which of the following best describes the mechanism of action of this drug? A. 5B-reductase inhibitor B. 5a-reductase inhibitor C. 4a-reductase inhibitor D. 5a-hydrolase inhibitor E. 4gamma-hydrolase inhibitor

B. 5a-reductase inhibitor

Case 1:A 15 year -old girl presents to the clinic with heavy menses lasting 9 days and occurring every three weeks. Her Menses started at age 13 . The menstrual history is recorded correctly as : A. 13X 21 X 15 B. 13X 21X9 C. 15X21X13 D. 15X13 X9

B. 13X 21X9

1. A 65-year-old man comes to the physician because of recurrent abdominal pain and blood in the stools for the past 2 months. Physical examination shows no abnormalities. A colonoscopy shows presence of a hemorrhagic exophytic mass growing into the lumen of the sigmoid colon. Biopsy from the mass shows malignant glands infiltrating the underlying stroma. A radical colectomy is performed, and the patient returns for a follow-up exam after 2 months. Which of the following gene mutations is most likely related to this patients condition? A. KIT B. APC C. ALK D. HER2 E. BRCA-1

B. APC KIT is malignant melanoma ALK is adenocarcinoma of the lung HER2 and BRCA1 is breast cancer (possibly ovarian cancer)

A 47-year-old man comes to the physician because of a 2-week history of mild fever, nausea, upper abdominal discomfort and dark urine. He has a history of intravenous drug use and was diagnosed with a Hepatitis B infection 3-months ago. His temperature is 39°C (102.2°F), pulse is 130/min, respirations are 20/min and blood pressure is 120/76 mmHg. Physical examination shows scleral icterus and right subcostal tenderness. His current serology report is shown. Which of the following is the most likely diagnosis? Positive - HBsAg, anti-HCV IgG, anti-HDV IgM, anti-HBc, anti-HBs IgM Negative - Anti-HBs A. Resolved Hepatitis C and vaccinated against Hepatitis B B. Acute Hepatitis B and co-infection with Hepatitis D C. Acute Hepatitis C and co-infection with Hepatitis B D. Acute Hepatitis C, resolved Hepatitis B and vaccinated against Hepatitis D E. Resolved Hepatitis C and resolved Hepatitis D

B. Acute Hepatitis B and co-infection with Hepatitis D

A 49-year-old woman taking ibuprofen for increasing joint pain in her hands presents with increasing pain in her midsternal area. Gastroscopy reveals multiple, scattered, punctate hemorrhagic areas in her gastric mucosa. Biopsies from one of these hemorrhagic lesions reveal mucosal erosions with edema and hemorrhage. No mucosal ulceration is seen. Which of the following is the most likely diagnosis? A. Active chronic gastritis B. Acute gastritis C. Autoimmune gastritis D. Chronic gastritis E. Peptic ulcer disease

B. Acute gastritis

A 75-year-old woman comes to the emergency department because of diarrhea for the past 3 days. She passed stool at least 5 times in the last 24 hours that has been watery with occasional blood. She has loss of appetite, malaise and abdominal pain. The patient was recently admitted to the hospital for a urinary tract infection and discharged on a course of ampicillin which she completed 2 weeks ago. Physical exam shows signs of dehydration and she has tenderness in the abdomen. Which of the following is the most important risk factor for her current condition? A. Age B. Antibiotic therapy C. Hospitalization D. Proton-pump inhibitor use E. UTI diagnosis

B. Antibiotic therapy

A 60-year-old woman comes to the physician because of an itchy, whitish lesion on the vulva. She attained menopause 5 years back. A physical examination shows multiple smooth whitish plaques over the vulva. Biopsy from the area shows hyperkeratosis, thinning of the epidermis, disappearance of the rete pegs and the dermis shows sclerosis and lymphocytic infiltrates. Which of the following is the likely etiopathogenesis of this condition? A. Friction induced lesion B. Autoimmunity C. Infection with HPV 11 D. Infection with streptococci E. Estrogen replacement therapy

B. Autoimmunity

8. A 36-year-old woman comes to the physician after noticing a lump in her right breast. Physical examination of the right breast shows a 4 cm firm, irregular, fixed mass in the upper outer quadrant breast. A fine needle aspiration of the right breast shows malignant clonal population of cells limited to ducts and lobules by the basement membrane, preserved myoepithelial cells. The mass is removed, and a sentinel lymph node dissection is performed. Which of the following findings will best predict a good prognosis for the patient? A. A. The patient has concurrent ductal carcinoma in situ in the same breast B. B. The sentinel node is negative for tumor C. C. The patient has a sister who had a similar type of breast cancer D. D. The tumor has a high grade E. E. Metastasize to axillary lymph node

B. B. The sentinel node is negative for tumor

10. A 39-year-old primigravida woman comes to the physician at 36 weeks of gestation because of a 2-day history of headache and swelling of the feet. Her temperature is 37°C (98.6°F), pulse is 80/min, blood pressure is 190/95 mm Hg and respirations are 18/min. Physical examination shows bilateral pitting pedal edema. Urinalysis shows 2+ proteinuria, but no blood, glucose, or ketones. Which of the following is the most likely pathogenesis of this condition? A. A. Atherosclerosis of medium-sized placental arteries B. B. Vasoconstriction of spiral arteries leading to placental ischemia C. C. Premature separation of placenta from the uterus D. D. Placental implantation into the myometrium E. E. Placental tissue occluding the internal os

B. B. Vasoconstriction of spiral arteries leading to placental ischemia

A 22-year- old man comes to the physician because of a 3-week history of progressively worsening lethargy, weakness and constipation. He has a 4-year history of GERD and usually takes an antacid for symptom relief. His antacid use has increased significantly over the last month with minimal resolution of symptoms. An abdominal ultrasound shows several small, non-obstructing renal calculi. Laboratory studies show; Sodium 145 mEq/L (N 135-145 mEq/L) Potassium 3.2 mEq/L (N 3.6-5.2 mmol/L) Chloride 102 mEq/L (N 96-106 mEq/L) Bicarbonate 34 mEq/L (N 20-30 mEq/L) Calcium 12.8 mg/dL (N 8.6-10.3 mg/dL) Which of the following is most likely an adverse effect of this medication? A. Diarrhea B. Belching C. Headaches D. Confusion E. Galactorrhea

B. Belching

2. A 57-year-old woman comes to the physician because of abdominal pain and bloating for the past 3-months. She says that her abdominal girth has increased during the same duration despite loss of appetite. She attained menopause 8 years ago and has been on hormone replacement therapy for the past 4 years. Pelvic examination shows a right adnexal mass. A CT scan of the abdomen shows ascites and a cystic mass arising from right ovary with evidence of omental involvement. A biopsy of the mass shows neoplastic epithelial cells with multilayering and nuclear atypia, infiltrating the ovarian stroma. Which of the following markers would most likely be helpful in monitoring the treatment of this patient? A. β-hCG B. CA 125 C. Follicle stimulating hormone D. Luteinizing hormone E. Inhibin

B. CA 125

1. A 65-year-old man comes to the physician because of recurrent abdominal pain and blood in the stools for the past 2 months. Physical examination shows no abnormalities. A colonoscopy shows presence of a hemorrhagic exophytic mass growing into the lumen of the sigmoid colon. Biopsy from the mass shows malignant glands infiltrating the underlying stroma. A radical colectomy is performed, and the patient returns for a follow-up exam after 2 months. Which of the following tumor markers will be helpful in identifying the recurrence of this malignancy? A. AFP B. CEA C. CA 19-9 D. PSA E. CA-125

B. CEA

*A-36-year-old woman comes to the physician because of a 1-week history of a swollen right knee. She had watery diarrhea and lower abdominal cramping 4 weeks ago. Her temperature is 38.5°C (101.3 °F), pulse is 80/min, respirations are 15/min and blood pressure is 121/80 mm Hg. Physical examination shows right knee swelling, warmth, and tenderness with pain on active and passive flexion. Which of the following three bacteria are most likely to be associated with her symptoms? A. Bacillus, Shigella, Clostridium B. Campylobacter, Salmonella, Shigella C. Clostridium, Escherichia coli, Shigella D. Escherichia, Helicobacter, Yersinia E. Salmonella, Shigella, Clostridium

B. Campylobacter, Salmonella, Shigella

A 22-year-old man comes to the emergency department because of a 2-day history of severe abdominal pain and bloody mucoid diarrhea. He states that he ate a "pink" chicken burger from a local restaurant 6 days ago. His temperature is 37.3oC (99.1oF), pulse is 87 /min, respirations are 15/min, and blood pressure is 115/75mmHg. Abdominal examination shows generalized tenderness with no rigidity or rebound tenderness. Laboratory studies show numerous leukocytes, RBCs and Gram negative, motile, microaerophilic, bacilli in the stool. Which of the following microbial characteristics best describes the causal agent? A. Spore forming B. Curved C. Tumbling motility D. Bi-polar staining E. Obligate aerobe

B. Curved

A 18-year-old obese college student comes to the physician because of a 2-month history of dyspepsia after large, spicy meals. In addition to lifestyle modifications, she is given an OTC magnesium hydroxide preparation for symptom relief. Which of the following adverse effects is most likely with this medication? A. Constipation B. Diarrhea C. Vomiting D. Headache E. Milk Alkalai Syndrome

B. Diarrhea

An 18-year-old obese college student comes to the physician because of a 2-month history of dyspepsia after large, spicy meals. In addition to lifestyle modifications, she is given an OTC magnesium hydroxide preparation for symptom relief. Which of the following adverse effects is most likely with this medication A. Constipation B. Diarrhea C. Vomiting D. Headache E. Milk Alkali Syndrome lol

B. Diarrhea

A 39-year-old woman comes to the physician because of a 4-week history of increased lethargy and decreased appetite. She also has a 3-week history of swelling in the feet and face. She has a 5-year history of type 2 diabetes mellitus. Her only current medication is metformin. Her temperature is 37⁰C (98⁰F), pulse is 70/min, respirations are 18/min and blood pressure is 138/88 mmHg. Physical examination shows facial swelling and pedal edema. Laboratory studies show serum creatinine:2.2mg/dL (n: 0.8-1.2mg/dL), Fasting plasma glucose:189 mg/dL and hemoglobin A1c: 10%. Which of the following is the next best step in the management of this patient? A. Increase the dose of metformin and add Pioglitazone B. Discontinue metformin and add insulin C. Discontinue metformin and add glyburide D. Increase the dose of metformin E. Discontinue the metformin

B. Discontinue metformin and add insulin

A 74-year-old man comes to the emergency room because of a sudden onset of pain in the left lower abdomen, which has been progressively worsening over the last 2 days. He states that the pain is unremitting. He has some diarrhea but no nausea or vomiting. He has no dysuria or hematuria. No weight loss. His temperature is 102°F. Bowel sounds are decreased. The patient has involuntary guarding. There is tenderness and rebound tenderness when the left lower quadrant is palpated. A fixed sausage-like mass is palpable in the area of tenderness. There is no costovertebral angle (CVA) tenderness. Rectal examination reveals brown stool, which is fecal occult blood test (FOBT) positive. Bloodwork demonstrates a leukocytosis. Which of the following is the most likely diagnosis? A. Colon cancer B. Diverticulitis C. Pancreatitis D. Pyelonephritis E. Appendicitis

B. Diverticulitis

A 37-year old woman comes to her physician because the waist size of her pants has been progressively increasing for the past 6 months. She is a nutritionist who eats a very balanced diet and exercises regularly. Her blood pressure is 124/80 mm Hg, pulse is 80/min, and temperature is 37C (98.6F). Physical examination shows a positive fluid wave. Paracentesis shows slight cloudy fluid that is positive for PAS staining. Abdominal ultrasound shows a cystic mass on the bilateral adnexal region with a normal uterus. The masses are surgically removed, and histological analysis shows signet ring cells. Which of the following genes is most likely associated with this condition? A. p53 B. E-cadherin C. MSH1 D. Rb E. WNT1

B. E-cadherin If you see signet rings in 1. Ovaries = Kruckenberg tumor from seeding of gastric tumors = Diffused adenal carcinoma 2. Breast = Lobular carcinoma in situ A will see in FAP AND HPV Pseudomyxoma peritonei seen in mucinous and appendiceal CA

***A 45-year-old woman comes to the physician because of a 5-month history of dyspepsia and progressive fatigue. She has lost 10-kg (22-lbs) of weight during the same period. She takes ibuprofen for premenstrual discomfort during her cycles. Her father died of an unidentified condition of the gastrointestinal tract. Physical examination shows conjunctival pallor and mild to moderate tenderness in the epigastric region. Microscopic examination of a biopsy specimen taken from the stomach mucosa shows sheets and clusters of mucin-filled cells with a signet-ring appearance. Which of the following etiologies most likely correlates with the pathogenesis of this condition? A. Chronic inflammation of the gastric mucosa B. E-cadherin gene mutation C. Persistent NSAID use D. Mutation in KIT gene E. Autoimmune destruction of gastric glands

B. E-cadherin gene mutation - Gastric adenocarcinoma C. Acute gastritis and ulcers D. GIST E. Atrophic auto immune gastritis

A 29-year-old pregnant woman in 2nd trimester comes to the physician because of a 15-month history of headaches, bitemporal hemianopia, depression, central obesity, bruising and red, puffy, rounded face. Laboratory studies show elevated ACTH. An MRI shows a pituitary adenoma. The patient is diagnosed with Cushing's disease. Surgery and radiotherapy are scheduled. While awaiting pituitary radiotherapy, the patient is prescribed a selective inhibitor of 11ß-hydroxylase. Which of the following is most likely an adverse effect associated with the drug administered to this patient? A. Hair loss B. Edema C. Hyperkalemia D. Hyperglycemia E. Anxiety

B. Edema

A 26-year-old woman comes to her gynaecologist because of amenorrhea for a period of 2 years. Her last menstrual cycle was 5 months ago. Vital signs are within normal limits. Physical examination shows acanthosis over the armpits and posterior neck, hirsutism and a BMI of 30. A pelvic ultrasound shows multiple cysts in right and left adnexa. Which of the following is likely seen on further laboratory studies? A. Hypoglycaemia B. Elevated estrone C. FSH:LH > 3 D. Elevated CA125 E. Decreased serum insulin

B. Elevated estrone (PCOS) CA125 elevated in ovarian cancers Insulin resistance = Increase insulin = Acanthosis nigricans and hyperglycemia Aromatase (CYP15) turns androgens to estrogen Post memo bleeding d/t exogenous estrogen = very high estrogen = a lot more LH and little FSH = no LH surge = no ovulation = infertility and amenorrhea Increased estrogen = granulosa cell tumor (CE bodies and coffee bean nuclei) and PCOS

A 32-year-old man comes to the physician because of a 4-week history of epigastric pain and bloating. He describes the pain as dull and crampy, grade 3-4/10 and worse right after a meal. His vital signs are all within normal limits. Physical examination shows mild epigastric tenderness. Laboratory investigation of fecal antigen tests for H. pylori are positive. Which of the following is the most likely diagnosis for this patient? A. GERD B. Gastric ulcer C. Duodenal ulcer D. Gastric cancer E. Esophagitis

B. Gastric ulcer

A 38-year-old woman comes to the physician because of a 3-week history of frequent urination, increased thirst and fatigue. She has no past medical history. She has a history of allergy to sulfa-containing drugs. Her temperature is 37.2⁰C (98.9⁰F), pulse is 68/min, respirations are 16/min and blood pressure is 122/82 mmHg. Physical examination shows no abnormalities. Laboratory studies show Fasting plasma glucose:210 mg/dL and hemoglobin A1c: 7.8%. Which of the following antidiabetic medications is most likely contraindicated in this patient? A. Metformin B. Glyburide C. Acarbose D. Exenatide E. Repaglinide

B. Glyburide

A 20-year old woman comes to the physician because of cauliflower like lesions on her vagina and constant vaginal itching. She is sexually active with multiple partners. She denies seeing a hard, painless lesion on her vagina. Her blood pressure is 120/80 mm Hg, pulse is 80/min and temperature is 37C (98.6F). Physical examination shows rough lesions surrounding the vulva. Biopsy of the lesions show large cells with a hyperchromatic nuclei and perinuclear halo. Which of the following etiologies is the most likely associated with this condition? A. HSV 2 B. HPV 6, 11 C. HPV 16, 18 D. Treponema pallidum E. HSV 1

B. HPV 6, 11 (low risk cancer)

A 30-year-old man comes to the physician because of a 3-month history of progressive dyspnea and fullness in the upper abdomen. His vital signs are within normal limits. Physical examination shows decreased breath sounds over both lower lung lobes and hepatomegaly. Serum studies show ALT 155 U/L (8-20 U/L) and AST 1700 (8-20 U/L). An x-ray of the chest shows hyperinflated lung fields and flattening of both domes of the diaphragm. A biopsy taken from the liver shows cytoplasmic inclusions within the hepatocytes which are PAS positive. This patient is at risk of developing which of the following most likely complications? A. Pancreatic carcinoma B. Hepatocellular carcinoma C. Primary sclerosing cholangitis D. Left ventricular hypertrophy E. Obstructive jaundice

B. Hepatocellular carcinoma - related to panacinar emphysema

4. A 34-year-old woman comes to the emergency department because of a 2-hour history of profuse vaginal bleeding and dizziness after giving birth at home. Her blood pressure is 76/55 mmHg, pulse is 132/min and respirations are 28/min. Physical examination shows a restless, diaphoretic woman with a uterine fundal height significantly above the umbilicus. Her clothes are blood soaked and blood is visible at the introitus. A nurse begins uterine massage while the physician administers a drug which acts via specific Gq protein coupled receptors leading to activation of myosin light chain kinase (MLCK) and myometrial contraction. Which of the following is most likely to occur at high concentrations of this drug? A. Hypernatremia B. Hyponatremia C. Ventricular arrhythmias D. Dehydration E. Constipation

B. Hyponatremia; drug which acts via specific Gq protein coupled receptors leading to activation of myosin light chain kinase (MLCK) and myometrial contraction = oxytocin, so at high concentrations you'll activate vasopressin (r.) —> fluid retention and water toxicity —> hyponatremia, seizure, shock or death

A 45-year-old man comes to the physician because of a 2-day history of nausea and 2 episodes of bloody vomiting. He is a chronic alcoholic and admits to a recent episode of binge drinking. He has smoked cigarettes occasionally over the past 5 years. Physical examination shows a slightly enlarged liver. An x-ray of the chest shows clear lung fields and normal cardiac silhouette. Upper GI endoscopy shows longitudinal mucosal tears at the gastroesophageal junction. Which of the following mechanisms best describes the etio-pathogenesis of this patient's condition? A. Loss of intrinsic inhibitory innervation of LES (Lower esophageal sphincter) B. Inadequate relaxation of LES during retching C. Engorgement of submucosal venous plexus D. Inflammation of the esophagus due to chronic irritation E. Incompetence of lower esophageal sphincter (LES)

B. Inadequate relaxation of LES during retching - MaLLory Weiss Syndrome (Longitudinal Lacerations of GE junction)

A 32-year-old man comes to the physician because of a 4-week history of epigastric pain and bloating. He describes the pain as dull and crampy, grade 3-4/10 and worse right after a meal. His vital signs are all within normal limits. Physical examination shows mild epigastric tenderness. The physician suspects that the patient may have a peptic ulcer. Which of the following is the most significant risk factor in the development of this condition? A. Alcohol B. Infection C. NSAID use D. Smoking E. Zollinger-Ellison syndrome

B. Infection

A 28-year-old man comes to the physician because of a 2-week history of abdominal pain, mild fever and malaise. About 6-months ago, he got a tattoo on his right arm while on a vacation in Peru, South America. His temperature is 38°C (100.4°F), pulse is 88/min and blood pressure is 120/80 mmHg. Physical examination shows pale yellow skin and mild scleral icterus. Laboratory studies show elevated AST and ALT levels. He is diagnosed with viral hepatitis. Which of the following is most likely mode of transmission of the causal pathogen? A. Ingestion B. Inoculation C. Congenital D. Inhalation E. Sexual

B. Inoculation

A 45-year-old man comes the physician because of progressive difficulty breathing and hoarseness. Physical examination shows an oval-shaped rubbery mass measuring 2x2cm in the thyroid gland. Radioactive iodine testing showed reduced uptake. Excisional biopsy shows well differentiated follicular cells with no mitotic figures. Which of the following morphological features would most likely be seen in this patient's condition? A. Anaplastic cells B. Intact capsule C. Capsular invasion D. Psamomma bodies E. Papillary growths

B. Intact capsule because it has no mitotic figures so it's benign and not malignant; cold nodule = mass that isn't making hormone

A 35-year-old man comes to the physician because of a 3-day history of fever, watery diarrhea, and abdominal pain. He works on a dairy farm and has a history of occasionally drinking unpasteurized milk. His temperature is 38.5°C (101.3 °F), pulse is 90/min, respirations are 15/min, and blood pressure is 121/80mmHg. Physical examination shows generalized abdominal tenderness without guarding. Stool analysis shows the presence of fecal leucocytes and the presence of Gram-positive rods, with tumbling motility. Which of the following bacterially encoded proteins is most likely responsible for the cellular uptake of the causal agent? A. Act A B. Internalin A C. Type IV pili D. Listeriolysin O E. Aggregative adherence fimbriae

B. Internalin A

A 37-year-old nulliparous and obese woman comes to the physician for a routine pap smear examination. Her last pap smear was 10 years ago. She has had multiple sexual partners over the last 10 years. Her mother was diagnosed with breast cancer at the age of 47. Her temperature is 37C (98.7F), blood pressure is 129/83 mmHg, pulse is 86/m and respirations are 20/m. Pap smear examination shows features of high grade squamous intraepithelial lesion. Which of the following is the most significant factor that predisposes this patient to develop these lesions? A. Nulliparity B. Lack of barrier contraception C. Obesity D. Early menarche E. Family history of Cancer

B. Lack of barrier contraception

A 25-year-old woman comes to the physician because of a 2-week history of abdominal pain, mild fever and malaise. She returned 3-weeks ago from a vacation to Bangkok, Thailand. While on vacation, she frequently consumed food at local street side vendors. She denies unprotected sexual intercourse or intravenous drug use. Her temperature is 38°C (100.4°F), pulse is 88/min and blood pressure is 120/80 mmHg. Physical examination shows hepatomegaly and mild scleral icterus. Laboratory studies show elevated AST and ALT levels. Which of the following is most likely associated with her condition? A. Brain B. Liver C. Kidney D. Lungs E. Ovary

B. Liver

9. A 30-year-old woman is brought to the emergency department by her boyfriend because of a 5-hour history of confusion, vomiting and abdominal pain. She has 15-year history of type-1 diabetes mellitus, but she is not compliant with her medications. Her blood pressure is 90/50 mm Hg, pulse is 110/min and respirations are 22/min. Physical examination shows dry skin and rapid, deep breathing. Laboratory studies show random blood glucose level of 350 mg/dL (N: <180> A. Normal pH and normal HCO3- levels B. Low pH and low HCO3- levels C. Low pH and high HCO3- levels D. High pH and normal HCO3- levels E. High pH and high HCO3- levels

B. Low pH and low HCO3- levels

2. A 32-year-old woman comes to the physician because of a 2-month history of heartburn and upper abdominal discomfort. She has been taking proton pump inhibitors once a day for the last 4 weeks. An upper Gl endoscopy with biopsy shows an ulcer on the antrum of the stomach. The biopsy of the ulcers is positive for the presence of gram negative, curvilinear, noninvasive and urease positive bacilli with intra-epithelial neutrophils. Which of the following is a likely complication of this condition? A. Pernicious anemia B. MALToma C. Fistula formation D. Pseudocyst E. Achalasia cardia

B. MALToma

3. A 45-year-old man comes to the physician because of a 2-month history of episodic headache, palpitations and abdominal discomfort. His pulse is 100/min, blood pressure is 160/90 mm Hg and respirations are 20/min. Physical examination shows no abnormalities. Laboratory studies show elevated plasma and urine metanephrines. A diagnosis of pheochromocytoma is suspected, however CT scan and MRI fail to identify any adrenal tumor. Which of the following additional investigations is most helpful in localizing the tumor? A. Ultrasound of the abdomen B. MIBG (Metaiodobenzylguanidine) scintigraphy C. Adrenal venous sampling D. Oral sodium loading test E. Inferior petrosal sinus sampling

B. MIBG (Metaiodobenzylguanidine) scintigraphy

A 24-year-old woman comes to the physician because of a one-day history of severe abdominal pain and watery stools 3 days after arriving in South America. Although she avoided salads and tap water, she drank some local passion fruit juice with crushed ice before becoming ill. Frequent episodes of diarrhea keep her confined to her hotel room. She has no underlying health problems and recovers 3 days later. Her recovery is uneventful, except for several days of fatigue. Laboratory studies show gram-negative, oxidase negative bacteria. Which of the following is the best agar to culture and identify the causal agent? A. Löwenstein-Jensen agar B. MacConkey Agar C. TCBS Agar D. buffered charcoal yeast extract (BCYE) agar E. Ashdown's agar

B. MacConkey Agar

A 40-year-old man comes to the physician because of a 3-day history of stiff jerky motions of his limbs which he cannot control. He has a 20-year history of type 2 diabetes mellitus. He has been recently diagnosed with diabetic gastroparesis and started treatment with the appropriate drug. Vitals are within normal limits. Physical examination shows peripheral neuropathy in a glove- stocking distribution. Which of the following is the most likely the medication given to this patient? A. Cisapride B. Metoclopramide C. Nizatidine D. Misoprostol E. Clarithromycin

B. Metoclopramide

A 38-year-old woman, gravida 1, para 0, at 22 weeks' gestation comes to the obstetrician because of a 5-day history of recurrent headaches. Her pulse is 86/min, respirations are 20/min, and blood pressure is 150/90 mm Hg. Physical examination shows bilateral pitting type of pedal edema. Laboratory investigations show proteinuria. Fetal ultrasonography shows a male fetus with no abnormalities. Which of the following is the most likely complication that can be seen in such a condition? A. Placenta accreta B. Microangiopathic hemolytic anemia C. Hydronephrosis D. Pseudomyxoma peritonei E. Meigs syndrome

B. Microangiopathic hemolytic anemia

A 35-year-old man who presents with a neck mass is found to have a serum calcium level of 11.8 mg/dL and periodic elevation of his blood pressure. Extensive workup reveals the presence of a medullary carcinoma of the thyroid, a pheochromocytoma, and hyperplasia of the parathyroid glands. Which of the following is the most likely diagnosis? A. Multiple endocrine neoplasia syndrome type 1 B. Multiple endocrine neoplasia syndrome type 2A C. Multiple endocrine neoplasia syndrome type 2B D. Polyglandular syndrome type I E. Polyglandular syndrome type II

B. Multiple endocrine neoplasia syndrome type 2A

A 46-year-old man comes to the physician because of a 4-day history of worsening pain on swallowing and burning sensation on his chest. He denies nausea or vomiting but describes excessive salivation. He was diagnosed 10 months ago with Leukemia, and he has been receiving chemotherapy treatments. Upper esophago-gastro-duodenoscopy (EGD) shows lesions supporting a diagnosis of infectious esophagitis caused by HSV-1. Which of the following was most likely observed during the endoscopy? A. Normal esophageal B. Multiple small ulcers less than 1.0cm diameter C. White patchy lesions that are easily dislodged D. Single oval ulcer of 2.5cm diameter E. Enlarged veins

B. Multiple small ulcers less than 1.0cm diameter

An 18-year-old man is brought to the emergency department because of severe head trauma sustained in a diving accident. Physical examination shows a decorticate, agitated young man who only opens his eyes in response to pain and makes incomprehensible sounds. He is intubated and admitted to the ICU. He is given medication as prophylaxis against stress related gastritis. Which of the following is most likely prescribed for this patient? A. Ondansetron B. Omeprazole C. Metoclopramide D. Cisapride E. Octreotide

B. Omeprazole

10. A 50-year-old woman comes to the physician because of a 1-day history of abdominal pain, nausea and vomiting. She was diagnosed with renal stones 4 months ago, for which she underwent treatment. She also has a history of right ulnar bone fracture 5 months ago, following a trivial fall. She is currently not taking any medications. Physical examination shows proximal muscle weakness. Serum studies show hypercalcemia, hypophosphatemia and elevated parathyroid hormone levels. Which of the following is the most likely cause of this patient's condition? A. Vitamin D deficiency B. Parathyroid adenoma C. Acute renal failure D. Medullary carcinoma of thyroid E. Multiple myeloma

B. Parathyroid adenoma

A 34-year-old man is brought to the emergency department by his wife because of 4-hour history of nausea and tremors. Physical examination shows abnormal sweating, difficulty speaking and lack of concentration. He recently changed his antidiabetic medications. Which of the following antidiabetic agents would be the most likely cause of the patient's symptoms? A. Alpha-glucosidase inhibitor B. Sulfonylurea C. Biguanide D. Thiazolidinedione E. DPP-4 inhibitor

B. Sulfonylurea - only one that causes hypoglycemia

A 47-year-old woman comes to the physician because of a 6-week history of a painless lump in the lower, outer quadrant of her left breast. She has no family history of breast, ovarian, tubal or peritoneal cancer. Physical examination shows slight dimpling of the skin over the lump which is firm, irregularly shaped and approximately 2 cm in diameter. Radiological findings are suggestive of malignancy. The histology report shows an estrogen receptor positive, invasive ductal carcinoma. After surgery, the patient is started on a drug that is an estrogen antagonist in breast and uterine tissue and an agonist in bone. Which of the following is the most likely prescribed drug? A. Tamoxifen B. Raloxifene C. Letrozole D. Clomiphene E. Fulvestrant

B. Raloxifene

A 60-year old woman comes to the physician because of a palpable mass in her right breast for the past 12 months. Physical examination shows a firm, non-tender 5 cm mass in the upper outer quadrant. There are no overlying skin changes, nipple retraction or axillary lymphadenopathy. Fine needle biopsy and microscopic examination of the mass shows a cellular stroma with surface slits and clefts protruding into spaces. The stromal cells showed mild atypia and no overgrowth with a mitotic rate of 2/HPF. Which of the following complications is most likely associated with this condition? A. Metastasis to axillary lymph nodes B. Recurrence after resection C. Malignant transformation D. Painful breast enlargement E. Bloody nipple discharge

B. Recurrence after resection (Phyllodes tumor so once you take it out it keeps coming back, so you have to get a double mastectomy); vignette says cellular stroma with surface slits and clefts protruding into spaces. The stromal cells showed mild atypia and no overgrowth with a mitotic rate of 2/HPF.

A 32-year-old man comes to the physician because of a 3-week history of anorexia, malaise, fatigue, passing dark urine and upper abdominal discomfort. He is homeless with a history of intravenous drug abuse. His temperature is 37.8°C (100°F). Physical examination shows scleral icterus, jaundice and liver is palpable 2 cm below the right costal margin. Laboratory studies show elevated liver enzymes and anti-HCV-IgM antibodies. Which of the following best characterizes the causal agent? A. Requires surface glycoproteins from HBV B. Replicates using RNA dependent RNA polymerase C. Contains negative sense, single stranded RNA within an envelope D. Infection can be prevented by a vaccine E. Replicates using RNA dependent DNA polymerase

B. Replicates using RNA dependent RNA polymerase

Which of the followings examination findings would suggest appendicitis? A. Murphy's sign B. Rovsing's sign C. Costovertebral tenderness D. Cullen sign

B. Rovsing's sign

5. A 26-year-old woman comes to the physician because of a 7-month history of a cyclical pain and a lump in her right breast. Although her discomfort is bearable most of the time, it increases significantly at the same phase of her menstrual cycle each month. She has no family history of breast cancer, exercises regularly and follows a healthy diet. Her vitals are within normal limits. Physical examination shows multiple firm, mobile, oval-shaped masses ranging in size from the <> A. Fulvestrant B. Clomiphene C. Danazol D. Mestranol E. Finasteride

C. Danazol ENDOCRINE drugs

A 25-year old Asian woman comes to the emergency department because of excessive vomiting and amenorrhea for the past 4 months. Her blood pressure is 140/90 mm Hg, pulse 90/min, respirations are 20/min, and temperature is 37C (98.6F). Physical examination shows an abdomen that is large for gestational age. A pregnancy test is positive. Laboratory studies show very high beta-hCG levels. An abdominal ultrasound shows a snowstorm appearance with no fetal tissue or detectable fetal heartbeat. Dilation and curettage shows trophoblastic proliferation with edematous villi. Which of the following will be karyotype of the mole? A. 69 XXY B. 69 XX C. 46 XY D. 45 XO E. 47XXY

C. 46 XY

A 30-year-old woman, G2P2, comes to the physician for her first prenatal examination. She is 10 weeks pregnant. However, she has experienced vaginal bleeding and abdominal pain over the last 3 weeks. Physical examination is consistent with approximately ten weeks of gestation. Laboratory studies show significantly elevated b-hCG levels. A transvaginal ultrasound of produces the image below. Dilatation and curettage shows grape like vesicles without any fetal parts. A karyotype of the excised tissue would most likely show which of the following? A. 45XO B. 46XX maternal C. 46XX paternal D. 69XXX, extra maternal set E. 69XYY, extra paternal set

C. 46XX paternal

3. A 25-year-old man comes to the physician because of 2-month history of recurrent episodes of rectal bleeding. His blood pressure is 110/70 mmHg, pulse is 90/min and respirations are 14/min. His uncle died of a colorectal cancer. Physical examination shows no abnormalities. Colonoscopy shows more than 100 polyps in the descending colon. Which of the following gene mutations is most likely associated with this condition? A. MSH2 B. Rb C. APC D. E-cadherin E. PTEN

C. APC

A 49-year-old woman presents with increasing problems swallowing food (progressive dysphagia). X-ray studies with contrast reveal that she has a markedly dilated esophagus above the level of the lower esophageal sphincter (LES). No lesions are seen within the lumen of the esophagus. Which of the following is the most likely cause of this disorder? A. Decreased LES resting pressure B. Absence of myenteric plexus in the body of esophagus C. Absence of myenteric plexus at the LES D. Absence of submucosal plexus in the body of esophagus E. Absence of submucosal plexus at the LES

C. Absence of myenteric plexus at the LES

A newborn infant is noted to have coughing and cyanosis during feeding. This infant is also noted to have marked gastric dilation due to "swallowed" air. Workup reveals that this infant has the most common type of esophageal atresia. Which one of the following statements correctly describes this type of congenital abnormality? A. Atresia of the esophagus with fistula between both segments and the trachea B. Atresia of the esophagus with fistula between the trachea and the blind upper segment C. Atresia of the esophagus with fistula between the trachea and the distal esophageal segment D. Atresia of the esophagus without tracheoesophageal fistula E. Fistula between a normal esophagus and the trachea

C. Atresia of the esophagus with fistula between the trachea and the distal esophageal segment

A 42-year-old man presents with weakness and dizziness associated with stress. Physical examination reveals a slightly decreased blood pressure along with a diffuse increase in skin pigmentation. Laboratory examination reveals hyponatremia and hyperkalemic acidosis with decreased aldosterone, decreased cortisol, decreased glucose, increased ACTH, decreased sex steroids, and increased LH and FSH. Thyroid function tests are found to be within normal limits. Which of the following is the most likely cause of this patient's signs and symptoms? A. A benign adenoma of the adrenal cortex B. A malignant tumor of the adrenal medulla C. Autoimmune destruction of the adrenal cortex D. Bilateral hyperplasia of the adrenal cortex E. Tuberculosis of the adrenal medulla

C. Autoimmune destruction of the adrenal cortex Primary adrenal insufficiency d/t autoimmune destruction of adrenal cortex, TB in third world countries, and N. meningitidus which leads to Waterhouse Friedrich Syndrome and leads to bilateral hyperplasia; so pit releases ACTH with increased skin pigmentation (MSH from POMC)

A 59-year-old man comes to the physician because of a 2-week history of lesion over the shaft of penis. He smokes 1 pack of cigarettes per day for the past 30 years. Physical examination shows an uncircumcised male with a painless ulcer over the ventral aspect of the shaft of the penis. The left inguinal lymphnodes are enlarged. Biopsy from the ulcer shows infiltrating neoplastic squamous cells with inter-cellular bridges and areas of necrosis. Which of the following predisposing factors is most likely implicated in this condition? A. Bowenoid papulosis B. Condyloma acuminatum C. Bowen disease D. Cryptorchidism E. Exposure to estrogen

C. Bowen disease

9. A 25-year-old woman comes to the physician because of a 6-month history of irregular menstruation and facial hair. Menarche occurred at 13 years, and menses occurred at regular 28-day cycles until 1 year ago. Her pulse is 82/min, blood pressure is 132/79 mm Hg and body mass index is 28 kg/m2. Secondary sexual characteristics are within normal limits. Laboratory studies show increased serum estrogen and androgen levels. Transvaginal ultrasonography shows the presence of multiple cysts in the right ovary. Which of the following additional laboratory findings is most likely in this patient? A. A. Low blood insulin levels B. B. High serum adrenocorticotropic hormone levels C. C. High blood glucose levels D. D. Low serum Prolactin hormone levels E. E. High serum levels of CA-125

C. C. High blood glucose levels

7. A 34-year-old woman comes to the physician because of a 4-day history of bloody discharge from the left breast nipple. She has regular menstrual cycles and is using oral contraceptives. Mammography of the breast shows calcification. A biopsy from the left breast nipple shows a delicate branching papilla in lumen with the fibrovascular core along with double layer of epithelial cells. Which of the following is the most likely diagnosis? A. A. Acute mastitis B. B. Fibroadenoma C. C. Intraductal papilloma D. D. Phyllodes tumor E. E. Sclerosing adenosis

C. C. Intraductal papilloma

A 66-year-old woman comes to the physician because of a 2-month history of vaginal spotting and lower abdominal pain. She attained menopause 7 years back. She has lost 8 kg (17.6 lb) of weight in the past 2 months. Pelvic examination shows no abnormalities. Pelvic imaging shows a large hemorrhagic mass in the uterine cavity. She is planned for a hysterectomy and histopathological examination of the resected specimen shows malignant neoplastic epithelial cells forming glands admixed with neoplastic rhabdomyocytes and extensive areas of necrosis and hemorrhage. Which of the following is the likely diagnosis? A. Endometrial polyp B. Endometrioid endometrial carcinoma C. Carcinosarcoma of the uterus D. Serous endometrial carcinoma E. Leiomyosarcoma

C. Carcinosarcoma of the uterus

9. A 48-year-old man comes to the physician because of a 2-day history of pain and swelling to the left scrotum. He also describes a preceding 5-day history of painful urination and an abnormal penile discharge. He reports unprotected sexual intercourse 2 weeks before his symptoms began. His temperature is 37.2°C (98.6°F), respirations are 16/min, pulse is 80/min and blood pressure is 126/75 mm Hg. Physical examination shows a mucoid penile discharge and unilateral scrotal swelling, erythema, and tenderness. Laboratory investigations on a discharge sample shows no growth on Modified Thayer Martin agar. Which of the following is the most likely diagnosis? A. Primary syphilis B. Gonorrhea C. Chlamydia D. Chancroid E. Donovanosis

C. Chlamydia

1. A 60-year-old man comes to the physician because of a 2-month history of progressive difficulty in swallowing solid foods and unintentional weight loss of 15-lbs. Physical examination shows severe emaciation with conjunctival pallor. Upper GI endoscopy shows a large exophytic mass growing into the lumen of the middle third of the esophagus. Microscopic examination of the biopsy from the mass shows neoplastic cells forming keratin pearls and invading the submucosa of the esophageal wall. Which of the following is an important known risk factor for this condition? A. HSV infection B. Gastro-esophageal reflux disease C. Cigarette smoking D. Diabetes mellitus E. Consumption of smoked food

C. Cigarette smoking

A 45-year-old man comes to the physician because of heartburn, regurgitation of a bitter-tasting fluid into his mouth and occasional nausea and vomiting. The physician prescribes a drug that can prevent both nausea and vomiting as well as promote upper gastrointestinal motility. Antagonism of which of the following receptors most likely contributes to the therapeutic effect of the drug? A. M3 cholinergic B. Nn cholinergic C. D2 dopaminergic D. H2 histaminergic E. Beta-2 adrenergic

C. D2 dopaminergic Want to shut off area postrema so you want to decrease serotonin = diarrhea and carcinoid syndrome; want to inhib 5HT3 antag, but also promotes gastric motility (erythromycin, cisapride, and metoclopramide). Metoclopramide is the best because it's a 4 aggy, 3 antaggy and doppy 2 antaggy which is bad cause you develop hyperprolactinemia (galactorrhea, decreased libido, infertility)

A 3-year-old girl is brought to the physician by her mother because of a 2- day history of watery diarrhea and fever. She has had 10 watery stools over the previous 24 hours, is fussy, and is refusing to eat. There is no history of vomiting. She attends a daycare 3 times per week. Her temperature is 38.5°C (101.3oF), pulse is 170/min, respirations are 25/min, and blood pressure is 80/56 mmHg. Physical examination shows an irritable and ill-appearing girl. Laboratory studies show motile Gram-negative bacilli and negative fecal lactoferrin test. She recovers fully with fluid replacement therapy. Which of the following mechanisms is most likely involved in the pathogenesis of the causal agent? A. Inhibition of adenylate cyclase B. Activation of guanylate cyclase C. Damage to small intestinal villi D. Loss of colonic epithelial cells E. Loss of cytoskeleton integrity

C. Damage to small intestinal villi -Ddx: Rotavirus, EPEC, ETEC, Norovirus •lab investigations tell us gram neg bacilli = EPEC and answer fits MOA

A 37-year-old woman comes to the emergency department because of a 2-hour history of mild, pleuritic chest pain and a syncopal episode that occurred 45 minutes ago. She denies shortness of breath. She has no other medical conditions and quit smoking 3 months ago. She takes a combined oral contraceptive pill and a multivitamin daily. Her blood pressure is 109/70 mmHg, pulse is 110/min, respirations are 26/min and temperature is 370C (98.6 F). Her physical examination is normal. She is hospitalized and treated for a pulmonary embolism. The physician advises her to change her current contraceptive pill and discusses options now available to her. Which of the following is the most likely option for this patient? A. Mestranol B. Ethinyl estradiol C. Desogestrel D. Clomiphene E. Tamoxifen

C. Desogestrel

A 40-year-old man comes to the physician because of a 2-day history of watery stools with no blood or mucus, abdominal cramps, and nausea. He is currently in Mexico. His temperature is 37.7°C (99.9°F), pulse 100/min, respirations 15/min, and blood pressure 110/70 mmHg. Physical examination shows a soft, non-tender, abdomen with hyperactive bowel sound. The diarrhea resolves in a couple of days without requiring antibiotic treatment. Laboratory studies show a gram-negative bacteria that increase the cGMP in the intestinal cells. Which of the following is the most likely causal agent? A. B. cereus B. Clostridium perfringens C. ETEC D. EPEC E. Vibrio cholerae

C. ETEC

A 9-year-old boy is brought to the physician by his mother because of a 5-day history of insomnia, restless sleep, and perianal itching. His siblings also have similar symptoms. His vital signs are all within normal limits. Physical examination shows mild perianal inflammation and several scratch marks. A Scotch tape test performed the following morning shows multiple transparent, oval, planoconvex eggs (see side image). Which of the following is the most likely causal pathogen? A. Entamoeba histolytica B. Giardia intestinalis C. Enterobius vermicularis D. Trichuris trichiura E. Ascaris lumbricoides

C. Enterobius vermicularis

Case 11: A 50 year old G0 LMP 1 week ago ago woman complains of increasing size of her abdomen and decreased appetite :Past Medical History :none. Meds none; FH : sister -Breast cancer SH: 2 wine on weekend .On inspection her abdomen is distended .What is highest in your differential diagnosis ? A. Pregnancy B. Germ Cell tumor of ovary C. Epithelial tumor of ovary D. Cirrhosis with ascites

C. Epithelial tumor of ovary - B more common in young people

2. A 45-year-old man is brought to the emergency department because of an episode of vomiting blood. He has a 20-year history of excessive alcohol consumption. Physical examination shows scleral icterus and ascites. Ultrasound of the abdomen shows features suggestive of liver cirrhosis. An upper GI endoscopy shows dilated and tortuous veins bulging into the lumen of the lower esophagus with overlying mucosal ulcerations. Which of the following diagnoses, best explains the hematemesis? A. Reflux esophagitis B. Mallory-Weiss syndrome C. Esophageal varices D. Esophageal webs E. Achalasia

C. Esophageal varices

5. A 45-year-old woman comes to the physician because of a 4-month history of a lump on the right side of her neck. She has not noticed any increase in the size of the lump. Physical examination shows cervical lymphadenopathy and a firm, painless mass in the right lobe of thyroid gland measuring 4-cm in diameter. A biopsy specimen obtained from the thyroid mass shows malignant cells characterized by the presence of papillae and calcifications. Which of the following risk factors is most likely responsible for the development of this patient's condition? A. Dyshormonogenesis B. Dietary iodine deficiency C. Exposure to ionizing radiation D. Antithyroglobulin antibodies E. Thyroid stimulating immunoglobulins

C. Exposure to ionizing radiation

A 35-year-old man comes to the physician because of 5-month history of an enlarging jaw, hands, and feet. He is also experiencing visual changes and fatigue. Laboratory studies show elevated IGF-1 and the patient is diagnosed with acromegaly. Which of the following drugs would be most effective in the treatment of this patient? A. Adrenocorticotropic hormone analog B. Vasopressin agonist C. Growth Hormone receptor antagonist D. GnRH Receptor Antagonists E. Gonadotropin-releasing hormone analog

C. Growth Hormone receptor antagonist

A 60-year-old woman comes to the physician because of 3 weeks history of painless vagina bleeding and lower abdominal pain. She has 5kg (11 lbs) unintentional weight loss over the last 2 month. She was diagnosed with vaginal intraepithelial neoplasia 5 years ago. Her temperature is 36.9C (98.4F), blood pressure is 156/82 mmHg, pulse is 90/m, respirations are 22/m. Physical examination shows bulky exophytic polypoidal mass. Histologic findings of the biopsy specimen shows polygonal squamous cells with keratin pearls. Which of the following is most likely risk factor for this condition? A. HPV 6 B. HPV11 C. HPV 18 D. HSV E. HIV

C. HPV 18

A 25-year old woman comes to the physician because of a 4-week history of abdominal pain, fever and malaise. She is an intravenous drug user since 6-months and often shares needles and syringes with 3 friends who have a similar lifestyle. Her temperature is 38°C (100.4°F), pulse 95/min and blood pressure is 114/76 mmHg. Physical examination shows hepatomegaly and mild scleral icterus. Laboratory studies show elevated AST and ALT levels. She is diagnosed with acute hepatitis C virus infection. Which of the following sequelae is most likely to occur as a result of her current condition? A. Hodgkin's lymphoma B. Cervical carcinoma C. Hepatocellular carcinoma D. Flaccid paralysis E. Acquired immunodeficiency syndrome

C. Hepatocellular carcinoma

A 25-year-old woman comes to the physician because of a 4-week history of abdominal pain, fever and malaise. She is an intravenous drug user since 6-months and often shares needles and syringes with 3 friends who have a similar lifestyle. Her temperature is 38°C (100.4°F), pulse 95/min and blood pressure is 114/76 mmHg. Physical examination shows hepatomegaly and mild scleral icterus. Laboratory studies show elevated AST and ALT levels. She is diagnosed with acute hepatitis C virus infection. Which of the following sequelae is most likely to occur as a result of her current condition? A. Hodgkin's lymphoma B. Cervical carcinoma C. Hepatocellular carcinoma D. Flaccid paralysis E. Acquired immunodeficiency syndrome

C. Hepatocellular carcinoma

12. A 28-year-old woman comes to the physician because of a 1-week history of several painless bumps on her genitalia. She initially did not pay attention to them, but now has noticed that they are gradually enlarging. Her temperature is 37.2°C (98.6°F), respirations are 16/min, pulse is 75/min and blood pressure is 129/75 mm Hg. Physical examination shows multiple small 'cauliflower' wart-like lesions on her genitalia. Laboratory studies show a circular dsDNA virus. Which of the following is the most likely causal agent? A. Herpes simplex Virus-2 B. Treponema pallidum C. Human Papillomavirus D. Cytomegalovirus E. Herpes Simplex Virus-1

C. Human Papillomavirus

A 26-year-old woman comes to the physician because of a 3-day history of pain and difficulty swallowing. She reports excessive salivation and inability to eat. She has a medical history of Leukemia and started chemotherapy 6 weeks ago. Endoscopic investigation shows multiple small ulcers at the distal 1/3 of her esophagus. Which of the following factors most likely contributed to the development of her infection? A. HSV-1 can have a latent infection B. Candida is part of the normal flora C. Immunosuppressed state D. Chemotherapy induce anemia E. IV antibiotics prophylaxis

C. Immunosuppressed state

A 47-year-old man presents with headaches, muscle weakness, and leg cramps. He is not currently taking any medications. Physical examination finds a thin adult man with mild hypertension. Laboratory examination reveals slightly increased sodium, decreased serum potassium level, and decreased hydrogen ion concentration. Serum glucose levels are within normal limits. A CT scan reveals a large tumor involving the cortex of his left adrenal gland. Which of the following combinations of serum laboratory findings is most likely to be present in this individual? A. Decreased aldosterone with increased renin B. Decreased cortisol with decreased ACTH C. Increased aldosterone with decreased renin D. Increased cortisol with increased ACTH E. Increased deoxycorticosterone with increased cortisol

C. Increased aldosterone with decreased renin with Conn Syndrome (Zona G controls RAAS and Zona F and R control ACTH) Aldosterone causes metabolic alkalosis with hypokalemia; metabolic acidosis has hyperkalemia; glucose is normal so cortisol is fine;

7. A 13-year-old girl is brought to the emergency department because of an 8-hour history of nausea, vomiting and drowsiness. Her temperature is 37°C (98.6°F), pulse is 110/min, blood pressure is 100/60 mm Hg and respirations are 28/min. Physical examination shows altered mental state, fruity odor of the breath and dry mucosal membranes. Laboratory studies show random blood glucose level of 450 mg/dL (Normal <200> A. Peripheral resistance to insulin action B. Amyloid deposition in the pancreas C. Increased protein and fat break down D. Decreased catecholamine secretion E. Decreased muscle break down

C. Increased protein and fat break down

A 67-year-old man comes to the emergency department because of shortness of breath and fever for the previous 2 days. He is admitted to hospital for intravenous antibiotic therapy. On day 4 of admission he complains of abdominal discomfort and the nurse notes that he has not passed stool in the past week. Docusate is prescribed. Which of the following best describes the mode of action of this drug? A. Binds water in the gut lumen B. Increases chloride secretion into the gut lumen C. Increases water absorption by the stool D. Irritates the gastric mucosa to stimulate peristalsis E. Stimulates the myenteric plexus

C. Increases water absorption by the stool

A 45-year-old man comes to the emergency department (ED) because of bloody diarrhea and abdominal pain for the past 2 days. He vomited 2 hours before coming to the ED. His Temperature is 38.5°C (101.3 °F), pulse is 90/min, respirations are 15/min, and blood pressure is 121/80mmHg. Physical examination shows generalized abdominal tenderness without guarding. Laboratory studies show a Gram negative, non-lactose fermenting, non-motile rod on stool analysis. Which of the following features best describes the causal agent? A. Halophilic B. Acid labile C. Infectious at low doses D. Tumbling motility E. Transmitted through aerosols

C. Infectious at low doses Salmonella is Acid Labile, Shigella is Acid Stable

*A 33-year-old man comes to the emergency department because of a 1-month history of intermittent fever, rigors and upper right abdominal pain. He emigrated from Brazil, South America, about 6-months ago. He frequently had intermittent bloody diarrhea in the past. His temperature is 38.4°C (101.1°F), pulse is 110/min and blood pressure is 120/74 mm Hg. Laboratory studies show elevated WBC (12x109/L) and elevated liver enzymes. A CT scan of his abdomen shows a cystic lesion in the right lobe of liver. A serology report shows antibodies to a protozoan parasite. Which of the following is the most likely mode of transmission of the causal pathogen? A. Consumption of undercooked meat B. Consumption of undercooked fish C. Ingestion of cysts in food/water D. Skin penetration by filariform larvae E. Sexual intercourse

C. Ingestion of cysts in food/water

A 33-year-old man comes to the emergency department because of a 1-month history of intermittent fever, and upper right abdominal pain. He emigrated from Brazil, South America, about 6-months ago. He occasionally had intermittent bloody diarrhea in the past. His temperature is 38.4°C (101.1°F), pulse is 98/min and blood pressure is 120/74 mm Hg. Laboratory studies show elevated WBC (12x109/L) and elevated liver enzymes. A CT scan of his abdomen shows a cystic lesion in the right lobe of the liver. A serology report shows antibodies to a protozoan parasite. Which of the following is the most likely mode of transmission of the causal pathogen? A. Consumption of undercooked meat B. Consumption of undercooked fish C. Ingestion of cysts in food/water D. Skin penetration by filariform larvae E. Sexual intercourse

C. Ingestion of cysts in food/water

A 44-year-old man comes to the physician because of a 1-month history of loss of appetite, abdominal discomfort and multiple episodes of non-bloody diarrhea. He recently emigrated from the Philippines, Southeast Asia. He has a history of intermittent non-bloody diarrhea. His temperature is 37°C (98.6°F), pulse is 130/min, respirations are 18/min and blood pressure is 110/76 mmHg. Physical examination shows hepatomegaly and mild jaundice. Laboratory studies show anemia, elevated liver enzymes and mild leukocytosis. An ova and parasite examination of his stool sample shows the presence of several oval eggs with an operculum. He is diagnosed with a trematode infection. Which of the following is most likely the mode of transmission of the causal pathogen? A. Skin penetration by larvae B. Insect bite C. Ingestion of undercooked freshwater fish D. Exposure to household pets E. Ingestion of undercooked meat

C. Ingestion of undercooked freshwater fish

A 28-year-old man comes to the physician because of a 6-month history profuse watery diarrhea and malaise. His stool frequency has increased from 4-5 episodes per day to 8-10 episodes per day. The stool is typically odorless and tea colored without blood or mucus. He also experiences abdominal discomfort and bloating. Physical examination shows an underweight adult with a slightly distended abdomen. An abdominal CT scan shows a neuroendocrine tumor indicative of a VIPoma. Lab results show; VIP (vasoactive intestinal peptide) 250 pg/mL (N <75)> A. Inhibition of acetylcholine release B. Increased gut peristalsis C. Inhibition of GI secretions D. Inhibition of 5HT3 receptors E. Inhibition of gut chloride channels

C. Inhibition of GI secretions

A 50-year-old man comes to the emergency department because of a 1-hour history of difficulty breathing. He was recently diagnosed with COPD and has been moderately well controlled on inhaled albuterol and beclomethasone as needed. Physical examination shows diffuse expiratory wheezes. He is started on appropriate pharmacotherapy and intravenous hydrocortisone is administered. Which of the following is most likely true regarding the medication given? A. It has a longer half-life B. It has a relatively long duration of action C. It has some salt-retaining effects D. It diffuses poorly across mucous membranes E. It is a synthetic corticosteroid

C. It has some salt-retaining effects

A 42-year-old man comes to the physician because of a 2-year history of headaches, bitemporal hemianopia, depression, central obesity, bruising and red, puffy, rounded face. Laboratory studies show elevated ACTH. An MRI shows a pituitary adenoma. The patient is diagnosed with Cushing's disease. Surgery and radiotherapy are scheduled. While awaiting surgery and pituitary radiotherapy, the patient is prescribed a non-selective inhibitor of steroid synthesis. Which of the following medications is most likely given to this patient? A. Hydrocortisone B. Spironolactone C. Ketoconazole D. Metyrapone

C. Ketoconazole

A 56-year-old man comes to the physician for a routine examination. He has a 6-year history of type 2 diabetes mellitus. His current medications are metformin and atorvastatin. His most recent HbA1c value was 7.1%. His blood pressure is 142/92 mmHg and pulse is 74/min. Laboratory studies show moderately increased albuminuria. All other studies are within the normal range. Which of the following drugs should be added to this patient's therapeutic regimen? A. Pioglitazone B. Atenolol C. Lisinopril D. Ezetimibe E. Amlodipine F. Insulin degludec

C. Lisinopril

A 45-year-old man comes to the physician because of 6-week history of increased thirst and frequent urination. He has a history of COPD, CHF, and hypertension. His current medications are ipratropium and lisinopril. His blood pressure is 135/87 mm Hg, pulse is 84/min, and BMI is 34.1 Kg/m2. Laboratory studies show a fasting blood glucose level of 195 mg/dL. He is diagnosed with type 2 diabetes mellitus. Which of the following antidiabetic drugs would be contraindicated in this patient? A. Sitagliptin B. Acarbose C. Metformin D. Repaglinide E. Glimepiride

C. Metformin contraindicated in hypoxia, renal and hepatic disease. COPD causes hypoxia and hyperemia —> respiratory alkalosis BAD Sitagliptin is a DD4 inhib which stop breakdown of your incretins. If you drink orange juice and inject IV glucose, you woul have more insulin so you breakdown incretins so you don't release endogenous insulin

12. A 10-month-old boy is brought to the physician by his mother because of a 24-hour history of diarrhea, and a 2-day history of low-grade fever of 37.7°C (100°F) and vomiting. His stools are watery and do not contain blood or mucus. His mother is unsure of the urine output, but he had a 0.7-kg (1.5-lb) weight loss in the past few days. His mother also says he is unusually irritable and has decreased activity levels since the onset of the illness. He has never been breastfed, and his vaccinations are not up to date. He attends a day-care center where two other children have the same symptoms. Which of the following virulence factors is most likely associated with his symptoms? A. Heat labile toxin B. Heat stable toxin C. NSP4 Protein D. Toxin B (TcdB) E. Urease enzyme

C. NSP4 Protein

A 45-year-old man comes to the emergency department because of a 2-day history of bloody diarrhea and abdominal pain. His temperature is 38.5°C (101.3 °F), pulse is 85/min, respirations are 16/min, and blood pressure is 125/80mmHg. Physical examination shows generalized abdominal tenderness without guarding. Laboratory studies show Gram negative, non-lactose fermenting, non-motile, rods on stool analysis. Which of the following microbial characteristics is most likely associated with the causal agent? A. Tumbling motility B. Microaerophilic C. Negative production of H2S D. Pink colonies of MacConkey's agar E. Pigmented colonies on EMB agar

C. Negative production of H2S

1. A 37-year-old woman comes to the emergency department because of a 2-hour history of mild pleuritic chest pain and a syncopal episode at home that occurred 45-minutes ago. She denies shortness of breath. She has no other medical conditions and quit smoking 3 months ago. She takes a combined oral contraceptive pill and a multivitamin daily. Her blood pressure is 109/70 mmHg, pulse is 110/min, respirations are 26/min and temperature is 37o C (98.6 F). Physical examination shows no abnormalities. She is hospitalized and treated for a pulmonary embolism. The physician advises her that she can no longer take the same type of contraceptive pill and discusses the options now available to her. Which of the following is the most likely the new drug prescribed for this patient? A. Mestranol B. Ethinyl estradiol C. Norgestimate D. Clomiphene E. Tamoxifen

C. Norgestimate

A 33-year-old man comes to the clinic because of a 4-hour history of epigastric pain, nausea, vomiting and loose, watery stool. He reports his last meal being half a breakfast burrito from yesterday which he left out of the refrigerator. His vital signs are within normal limits. Physical examination shows slightly dry mucosal membranes. Laboratory studies of a stool sample is negative for blood. Which of the following is the next best step in management for this patient? A. Endoscopy and biopsy B. Oral antibiotics C. ORS and hydration D. IV antibiotics E. Urease breath test

C. ORS and hydration

1. A 40-year-old woman comes to the physician because of a 10-month history of right upper quadrant abdominal pain and progressively worsening shortness of breath. She has never smoked but consumes 1-2 beers occasionally. Physical examination shows wheezing, hyper-resonant percussion tones in the lower lung fields and mildly tender hepatomegaly. An x-ray of the chest shows increased antero-posterior diameter with hyperinflated lungs. Lung function tests show findings consistent with obstructive lung disease. Serum studies show elevated levels of liver enzymes. Which of the following investigations can be most helpful to diagnose this condition? A. Endoscopy B. ERCP C. PAS-stained liver biopsy D. Serology E. Echocardiography

C. PAS-stained liver biopsy

A 25-year-old man comes to the physician because of a 4-week history of abdominal pain, fever and malaise. He is an intravenous drug user since 6-months and often shares needles and syringes with 3 friends who have a similar lifestyle. His temperature is 38°C (100.4°F), pulse 95/min and blood pressure is 114/76 mmHg. Physical examination shows hepatomegaly and mild scleral icterus. Laboratory studies show elevated AST and ALT levels. He is diagnosed with acute hepatitis C virus infection. A serology report of this person is most likely to show which of the following? A. Presence of IgA antibodies B. Absence of IgM antibodies C. Presence of IgM antibodies D. Presence of IgG antibodies E. Absence of IgA antibodies

C. Presence of IgM antibodies

3. A 22-year-old man comes to the physician because of a 5-day history of fever, lethargy, lower abdominal pain and bloody, mucoid diarrhea. He has a history of similar episodes of diarrhea in the past two years. Physical examination shows conjunctival pallor, finger clubbing and tenderness over the left iliac fossa. A colonoscopy shows mucosal inflammation with a continuous broad-based ulcer in the descending colon and rectum, containing islands of multiple polypoid mucosal growths. A biopsy from the lesion shows dense leukocytic infiltration of the mucosa with multiple crypt abscesses. Which of the following conditions can be associated with this disease? A. Pernicious anemia B. Autoimmune hepatitis C. Primary sclerosing cholangitis D. Liver abscess E. Perforation peritonitis

C. Primary sclerosing cholangitis

*A 58-year-old man comes to the physician because of a 10-day history of mild fever, nausea, upper abdominal discomfort and dark urine. He returned 4-weeks ago from vacation to Mexico. His temperature is 38°C (100.4°F). Physical examination shows jaundice, scleral icterus and right upper quadrant tenderness. Laboratory studies show hyperbilirubinemia, elevated liver aminotransferases and a 1:1250 anti-HAV-IgM titer. Which of the following features best characterizes the causal agent? A. Utilizes reverse transcriptase for replication B. Causes hepatocellular carcinoma C. Produces a polyprotein during translation of genome D. Produces a large amount of non-infectious particles E. Requires an envelope protein for attachment to host cells

C. Produces a polyprotein during translation of genome

1. A 40-year-old alcoholic woman comes to the physician because of a 1-week history of itching, yellow discoloration of the eyes, abdominal pain and swelling of the feet. Her blood pressure is 110/70 mm Hg, pulse is 95/min and temperature is 37°C(98.6°F). Physical examination shows palmar erythema, finger clubbing and pedal edema. Laboratory studies shows an AST/ALT ratio > 2. Ultrasound of the abdomen shows a shrunken liver. Which of the following additional findings is most likely seen in this patient? A. Gallstones B. Right sided heart failure C. Protosystemic shunting D. Elevated levels of Albumin E. Decreased GGT

C. Protosystemic shunting

1. A 55-year-old woman comes to the physician because of a 1-month history of itching, yellow discoloration of the eyes, abdominal pain and swelling of the feet. She drinks 60-ml of whiskey every day for the past 25 years. Her blood pressure is 110/70 mm Hg, pulse is 95/min and temperature is 37°C (98.6°F). Physical examination shows palmar erythema, finger clubbing and pedal edema. Laboratory studies show decreased serum albumin and mildly elevated levels of AST and ALT. Ultrasound of the abdomen shows a shrunken liver. Which of the following histological findings are most likely to be seen in this patient's liver? A. Microvesicular fatty change with perivenular fibrosis B. Fulminant hepatitis C. Regenerative hepatocyte nodules with bridging fibrous septa D. Dense inflammatory infiltrate in the interface region E. Ballooning of the hepatocytes

C. Regenerative hepatocyte nodules with bridging fibrous septa

1. A 40-year-old woman comes to the physician because of a 10-month history of right upper quadrant abdominal pain and progressively worsening shortness of breath. She has never smoked but consumes 1-2 beers occasionally. Physical examination shows wheezing, hyper-resonant percussion tones in the lower lung fields and mildly tender hepatomegaly. An x-ray of the chest shows increased antero-posterior diameter with hyperinflated lungs. Lung function tests show findings consistent with obstructive lung disease. Serum studies show elevated levels of liver enzymes. Which of the following mechanisms best explain the etio-pathogenesis of this condition? A. Accumulation of endogenous metal in liver B. Immunological destruction of the biliary tree C. Retention of the mutant protein in hepatocyte D. Virus induced apoptosis in the hepatocytes E. Alcohol induced cytoskeletal damage

C. Retention of the mutant protein in hepatocyte

A 27-year-old man comes to the physician because of a 1-week history of rash on his palms and soles. He is sexually active with his girlfriend that he met 3-months ago. His temperature is 37.2°C (99°F), respirations are 16/min, pulse is 80/min and blood pressure is 121/75 mmHg. Physical examination shows reddish-brown macular lesions on his palms and soles. Laboratory studies shows a positive screening test. Which of the following is the most likely diagnosis? A. Tertiary syphilis B. Primary syphilis C. Secondary syphilis D. Latent syphilis E. Acute meningococcemia

C. Secondary syphilis

A 2-week-old neonate presents with regurgitation and persistent, severe projectile vomiting. An olive-like epigastric mass is felt during physical examination. A chest x-ray does not reveal the presence of bowel gas in the chest cavity. This infant's mother did not have polyhydramnios during this pregnancy. Which of the following is the most appropriate treatment for this infant's condition? A. Oral medication with omeprazole and clarithromycin B. Oral medication with vancomycin or metronidazole C. Surgery to cut a hypertrophied stenotic band at the pylorus D. Surgery to remove a mass of the adrenal gland E. Surgery to resect an aganglionic section of the intestines

C. Surgery to cut a hypertrophied stenotic band at the pylorus

1. A 35-year-old woman comes to the physician because of a 1-month history of progressive fatigue and cold intolerance. Her pulse is 70/min, blood pressure is 100/80 mm Hg and respirations are 18/min. Physical examination shows cold, dry skin, hyporeflexia and enlargement of the thyroid gland. Thyroid function tests show increased TSH, decreased T3 and T4 levels and presence of thyroid peroxidase antibodies in the serum. This patient has an increased risk of developing which of the following complications? A. Thyrotoxicosis B. Osteoporosis C. Thyroid B-cell lymphoma D. Mental retardation E. Umbilical hernia

C. Thyroid B-cell lymphoma

A 24-year-old woman comes to the physician because of a 3-day history of abnormal vaginal discharge and painful urination. She currently lives with her new boyfriend. Her temperature is 99°F (37.2°C), respirations are 16/min, pulse is 80/min and blood pressure is 121/75 mm Hg. Physical examination shows thin, green, malodorous discharge and diffuse cervical erythema and petechiae. Wet mount examination shows motile flagellated protozoa. Which of the following is the most likely diagnosis? A. Bacterial Vaginosis B. Vulvovaginal Candidiasis C. Trichomoniasis D. Gonorrhea E. Chlamydia

C. Trichomoniasis

A 56-year old man comes to the emergency department because of a 1-month history of intermittent fever, rigors and upper right abdominal pain. He recently emigrated from Vietnam, Southeast Asia. He frequently had intermittent bloody diarrhea in the past. His temperature is 38.4°C (101.1°F), heart rate is 110 beats/min and blood pressure is 120/74 mm Hg. Laboratory studies show elevated WBC (12x109/L), ESR at 72 mm/h and elevated liver enzymes. A CT scan of his abdomen shows a cystic lesion in the right lobe of liver (see image). A serology report confirms the diagnosis. Which of the following features best characterizes the causal agent A. Trophozoites with cilia B. Oval eggs with operculum C. Trophozoites with pseudopodia D. Filariform larvae E. Sporozoite

C. Trophozoites with pseudopodia

2. A 42-year-old man comes to the physician because of 2-month history of abdominal discomfort and altered bowel habits. His blood pressure is 100/70 mm Hg, pulse is 112/min and respirations are 14/min. Physical examination shows no abnormalities. Colonoscopy shows several polyps in the descending colon. Some of the polyps are excised for histological examination. Which of the following types of polyps has the highest likelihood to undergo malignant transformation? A. Hyperplastic polyp B. Tubular adenoma C. Villous adenoma D. Pseudopolyp E. Inflammatory polyp

C. Villous adenoma

A 34-year-old woman comes to the physician because of a 4-day history of vaginal itching and burning. She has type 1 diabetes. Her vitals are temperature 37.2°C (99°F), respirations 16/min, pulse 80/min and blood pressure 121/75 mm Hg. Physical examination shows a white-yellow, curd like discharge on the vaginal walls and labial irritation. Laboratory investigation of the discharge shows budding yeasts and pseudohyphea. Which of the following is the most likely diagnosis? A. Bacterial vaginosis B. Trichomoniasis C. Vulvovaginal candidiasis D. Gonorrhea E. Chlamydia

C. Vulvovaginal candidiasis

3. A 20-year-old woman comes to the physician because of pain and fullness in the lower abdomen for the past 3 months. Physical examination shows a palpable right lower abdominal mass. Laboratory studies show elevated levels of serum alpha fetoprotein and α-1 antitrypsin. Ultrasound of the pelvis shows presence of large right ovarian mass. Excision biopsy of the mass shows layers of epithelial cells surrounding blood vessels forming Schiller Duval bodies. Which of the following is the most likely diagnosis of this condition? A. Brenner's tumor B. Dysgerminoma C. Yolk-sac tumor D. Fibrothecoma E. Krukenberg tumor

C. Yolk-sac tumor

2. A 4-month-old girl is brought to the physician by her mother because of ambiguous external genitalia. Her temperature is 37°C (98.6°F), pulse is 122/min, blood pressure is 140/90 mm Hg and respirations are 26/min. Physical examination shows clitoromegaly and pubic hair. Laboratory studies show hyponatremia, metabolic alkalosis, decreased plasma angiotensin-II activity and increased serum androgen levels. A CT scan of the abdomen shows bilaterally enlarged adrenal glands. Increase in which of the following is the most likely cause of hypernatremia in this patient? A. 17-hydroxy progesterone B. Testosterone C. Aldosterone D. 11-deoxycorticosterone E. Glucocorticoid production

D. 11-deoxycorticosterone

An XX infant is found to have external male genitalia and internal female genitalia. Physical examination reveals decreased blood pressure, while laboratory examination reveals a serum sodium level of 132 meq/L. Additionally, bilateral adrenal cortical hyperplasia is present. A deficiency of which of the following enzymes is most likely to produce the clinical findings in this infant? A. 3-β-dehydrogenase B. 11-hydroxylase C. 17-hydroxylase D. 21-hydroxylase E. 1-α-hydroxylase

D. 21-hydroxylase

A 28-year-old man comes to the physician because of a 6-week history of dry itchy skin and bruises that take too long to heal. He has a family history of type 2 diabetes mellitus.. His temperature is 37.2⁰C (98.9⁰F), pulse is 74/min, respirations are 15/min and blood pressure is 126/86 mmHg. Physical examination shows no abnormalities. Laboratory studies show Fasting plasma glucose:222 mg/dL and hemoglobin A1c: 8.0%. The patient is advised about lifestyle modifications and appropriate pharmacotherapy is started. Which of the following best describes the mechanism of action of the most appropriate antidiabetic drug for this patient? A. Inhibition of SGLT2 receptors B. Activation of PPAR-gamma receptors C. Closure of Potassium channels on β cells D. Activation of AMP-K E. Activation of GLP-1 receptors

D. Activation of AMP-K

***A 50-year-old man comes to the emergency department because of a 24-hour history of profuse, watery diarrhea and abdominal pain. His stools appear cloudy with flecks of mucus. He was on a business trip to Southeast Asia in the last 5 days. His temperature is 37o C 98.6oF), pulse is 120/min, respirations are 15/min and blood pressure is 100/60 mmHg. Physical examination shows dry oral mucous membranes, and capillary refill of 3 seconds. Laboratory studies of stool shows motile Gram -negative bacilli that produces heat labile enterotoxins (LT). Which of the following best describes the action of the toxin involved? A. Inhibition of guanylate cyclase B. Inhibition of adenylate cyclase C. Inhibition of 28S RNA on 60S ribosome D. Activation of adenylate cyclase E. Activation of guanylate cyclase

D. Activation of adenylate cyclase - ETEC, Vibrio

A 58-year-old man is admitted to the emergency department because of fever, abdominal pain, yellowish skin and altered consciousness. He is homeless and has a past history of intravenous drug use. His temperature is 38°C (100.4°F). Physical examination shows jaundice, scleral icterus and right upper quadrant tenderness. Laboratory studies show abnormal liver function. Serology studies for HBsAg are positive. He is diagnosed with acute fulminant hepatitis. Which of the following additional serology findings is most likely to contribute to the diagnosis? A. Anti-HDV-IgG B. Anti-HBs C. Anti-HBc D. Anti-HDV-IgM E. Anti-HAV-IgG

D. Anti-HDV-IgM

A 29-year-old man comes to the physician because of a 1-month history of abdominal pain, diarrhea and weight loss. His blood pressure is 125/82 mm Hg, respirations are 18/min, pulse is 100/min and temperature is 37.8°C (100°F). Physical examination shows a young male in painful distress with right lower quadrant tenderness. After the appropriate investigations, the patient is diagnosed with ulcerative colitis . He is given high doses of oral prednisone to treat his flare. Which of the following adverse effects is associated with this medication if given long term? A. Hemolysis B. Black stools C. Stevens-Johnson syndrome D. Avascular necrosis of the hip E. Extrapyramidal symptoms

D. Avascular necrosis of the hip

A 25-year-old man comes to the physician because of a 2-day history of profuse watery stools, and abdominal cramps. He denies any blood or mucus in the stool. He returned 4 days ago from a tour to Africa, and one of the cities he visited has an ongoing outbreak of a similar illness. His temperature is 37.7°C (99.9°F), pulse rate is 110/min, respirations are 15/min, and blood pressure is 100/60 mmHg. Laboratory studies show an organism that ferments sucrose on TCBS agar. Which of the following virulence factors is most likely related to this patient symptoms? A. Urease production B. Plasmid encoded toxin C. Capsule D. Bacteriophage encoded toxin E. Anaerobic, spore-former

D. Bacteriophage encoded toxin

3. A 32-year-old man come to the physician because of a 4-month history of epigastric pain and increased fatigability. He had an episode of hematemesis 1 day ago. An upper endoscopy is performed which shows an ulcerative lesion with heaped up margins in the body of the stomach. A biopsy of the lesion shows presence of neoplastic glands invading the lamina propria. Which of the follow etiologies is most likely linked to this process? A. Antibodies against the parietal cells B. Prolonged use of painkillers C. Vagal stimulation D. CDH 1 gene mutation E. Achalasia

D. CDH 1 gene mutation

A 40-year-old man comes to the physician because of 2-week history of fever and dysuria. He has a history of type 2 diabetes mellitus. His blood pressure is 126/74 mm Hg, pulse is 70/min, respirations are 14/min and temperature is 99.1°F (37.3°C). Urinalysis shows high urine sodium and glucose. Which of the following antidiabetic agents was this patient most likely prescribed? A. Metformin B. Pioglitazone C. Chlorpropamide D. Canagliflozin E. Acarbose

D. Canagliflozin because it's a SGLT-2 inhibitor which causes hyponatremia and hypoglycemia —> leads to UTI METFORMIN - Biguanide causing AMPK activation and inhibits gluconeogenesis, AE includes lactic acidosis, following hypoxemia = respiratory acidosis —> very very low pH, and renal/hepatic insuff PIOGLITAZONE (Glita = glitter at the party or PPARy ) weight gain and edema; constraint in patients with CHF; improve lipid profile CHLOPROPAMIDE - 1st gen SULFONYLUREAS which are bad cause they're the first, but sticks to SUR1 and inhib ATP-sensitive K+ channels in B-cells to keep K inside the cell leading to depol and and then Ca influx which releases insulin; causes weight gain and hypoglycemia with SIADH —> uvolemic hypernatremia and hyperemic flushing ACARBOSE inhib a-glucosidase help build up di and trisacchrides as osmotic gradients in stool and pulls water into it so there's diarrhea and flatulence; constraindic in IBS and IBD patients and those with renal impairment

A 25-year-old woman comes to the physician because of diarrhea and abdominal cramping for 2-days. She says that the diarrhea is foul-smelling and she feels bloated. She recently went hiking with her friends. A micrograph from a stool sample is shown. Which of the following is the most likely route of acquisition of the parasite? A. Consumption of preformed toxin in food B. Penetration of skin C. Consumption of wild forest plants D. Consumption of contaminated water E. Consumption of contaminated canned foods

D. Consumption of contaminated water, like lake or pond, or Forrest ranger —> steatorehea

A 45-year-old woman comes to the physician because of a 10-week history of fever, abdominal pain, and anorexia. The pain is grade 4-5/10 and localized to the right upper abdomen without radiations. About 6-months ago, she immigrated from India. Her temperature is 39°C (102.2°F), pulse is 124/min, respirations are 19/min and blood pressure is 110/72 mmHg. Laboratory studies show mild anemia, leukocytosis and elevated liver enzymes. An abdominal CT shows a 5x5 cm lesion in the right lobe of the liver. Needle aspiration and analysis of the abscess fluid shows the presence of trophozoites. She is diagnosed with an amoebic liver abscess. Which of the following is the most likely route of acquisition of the causal pathogen? A. Walking barefoot B. Insect bite C. Sexual activity D. Contaminated food/water E. Respiratory aerosols

D. Contaminated food/water

6. A 21-year-old woman comes to the physician because of a 2- month history of missing menses. She is not sexually active. Physical examination shows an obese woman with hirsutism. Pelvic examination shows a normal sized uterus, with a prominent adnexal mass. Pelvic ultrasound shows an enlarged ovary with multiple cysts. Which of the following is the most likely diagnosis? A. A. Immature teratomas B. B. Krukenberg tumors C. C. Ovarian cystadenocarcinomas D. D. Polycystic ovarian syndrome E. E. Tubo-ovarian abscesses

D. D. Polycystic ovarian syndrome

3. A 35-year-old man comes to the physician because of a 2-month history of yellowish discoloration of the skin and progressive abdominal discomfort. His vitals are within normal limits. Physical examination shows scleral icterus and hepatomegaly. Laboratory studies show increased levels of serum ferritin and biopsy taken from the liver is positive for Prussian blue stain. Which of the following is the most likely complication this patient can develop? A. Plummer vinson syndrome B. Aplastic crisis C. Cholangiocarcinoma D. Diabetes mellitus E. Emphysema

D. Diabetes mellitus

3. A 23-year-old man comes to the physician because of a 1-month history of fatigue, lethargy and recurrent episodes of diarrhea. He describes the stools as pale, bulky, foul smelling and difficult to flush. Physical examination shows tachycardia and conjunctival pallor. Laboratory studies show low serum iron levels and anti-endomysial antibodies. An endoscopic biopsy from the proximal small intestine shows villous atrophy and elongated hyperplastic crypts. Which of the following interventions would most likely improve this patient's condition? A. Surgical resection B. Antibiotic therapy C. Radiation therapy D. Diet modification E. NSAID therapy

D. Diet modification

A 35-year-old man comes to the physician because of a 2-month history of heaviness in the left side of the scrotum. He says that he had an undescended testis on the left, which was surgically corrected at 12-months of age. Physical examination shows a non-tender, firm swelling over the left testis which does not transilluminate. Imaging studies confirm a diagnosis of a testicular tumor. The patient is planned for surgical excision of the left testis. Histopathological examination of the tumor shows a homogenous, well-circumscribed mass composed of large, round, uniform tumor cells with a clear cytoplasm, prominent nucleoli and sparse lymphocytic infiltrates in the stroma. There were no areas of necrosis or hemorrhage within the tumor. Which of the following tumors in women is histologically similar to this patient's tumor? A. Ovarian choriocarcinoma B. Brenner tumor C. Ovarian chocolate cysts D. Dysgerminoma E. Ovarian yolk sac tumor

D. Dysgerminoma

A 45-year-old man comes to the emergency department (ED) because of a 2-day history of bloody diarrhea and abdominal pain. His Temperature is 38.5°C (101.3 °F), pulse is 85/min, respirations are 16/min, and blood pressure is 125/80mmHg. Physical examination shows generalized abdominal tenderness without guarding. Laboratory studies show a Gram negative, lactose fermenting, motile, rod on stool analysis. Which of the following is most likely causal agent? A. Salmonella Enteritidis B. Shigella sonnei C. Clostridium perfringens D. Enteroinvasive Escherichia coli E. Listeria monocytogenes

D. Enteroinvasive Escherichia coli

A 62-year-old woman comes to the physician because of a 2-month history of bloating, nausea, vomiting, weight loss of 18lbs and "early feelings of fullness". She has a 30-year history of hypertension and type 2 diabetes mellitus. She has been admitted to hospital 3 times within the last month due to erratic blood sugar readings. The last admission was one-week ago because of tachycardia, tachypnea, confusion and disorientation. Which of the following drugs, is most likely contraindicated in this patient? A. Repaglinide B. Glipizide C. Insulin D. Exenatide E. Metformin

D. Exenatide

2. A 69-year-old man comes to the physician because of recurrent abdominal pain for the past 6 months. Physical examination shows conjunctival pallor. An upper GI endoscopy shows a mucosal ulcer in the antrum with heaped up margins. A biopsy from the base of the ulcer shows malignant glands invading the underlying stroma. Which of the following is the most likely risk factor? A. Aflatoxin exposure B. HPV infection C. NSAIDs use D. H. pylori infection E. E-cadherin mutation

D. H. pylori infection

1. A 65-year-old man comes to the physician for a routine health checkup. He has smoked 2-packs of cigarettes per day for the past 30 years. HEENT examination shows a well-defined white patch over the buccal mucosa that could not be easily scraped off. A biopsy of the lesion shows epidermal thickening, hyperkeratosis and hyperchromasia with intact basement membrane. Other than tobacco use, which of the following infections is the most likely risk factor for this condition? A. HIV B. EBV C. H. pylori D. HPV E. HBV

D. HPV

A 57-year-old comes to the physician because of a 3-week history of fever, abdominal pain, nausea and vomiting. He was diagnosed with HIV 2-years ago and is on antiretroviral therapy. His temperature is 39°C (102.2°F), pulse is 130/min, respirations are 18/min and blood pressure is 110/76 mmHg. Physical examination shows scleral icterus and right upper quadrant tenderness. Laboratory studies show a CD4 count of 950 cells/mm3 and elevated levels of bilirubin, liver enzymes and prothrombin time. A qRT-PCR analysis shows an HIV load of 7, 200 copies/mL and the presence of an infectious agent containing a positive sense RNA genome within an envelope. He is most likely co-infected with which of the following viruses? A. Hepatitis A virus B. Hepatitis B virus C. Adenovirus D. Hepatitis C virus E. Cytomegalovirus

D. Hepatitis C virus

A 54-year-old woman comes to the physician because of heat intolerance, nervousness, chest palpitations, and weight loss, despite increased appetite. Her blood pressure is 180/98 mm Hg, pulse is 134/min, and temperature is 37.4 ºC (99.3 ºF). An ECG shows sinus tachycardia, with no signs of ST segment changes. Hyperthyroidism is suspected, and blood tests are requested. In addition to the presence of thyroid-stimulating antibody (TSAb), which of the following sets of results would confirm the diagnosis of Graves' disease in this patient? A. High T4, high TSH B. Low T4, high TSH C. Low T4, low TSH D. High T4, low TSH E. Normal T4, low TSH

D. High T4, low TSH

A 36-year-old woman is brought to the emergency department because of a 2-hour history of confusion and reduced consciousness. Her blood pressure is 110/70 mm Hg, pulse is 96/min and respirations are 16/min. Physical examination shows altered mental status, and yellowish discoloration of the skin and sclerae. A slit lamp examination shows corneal Kayser-Fleischer rings. Laboratory studies show elevated levels of serum bilirubin, low levels of serum albumin and low levels of serum ceruloplasmin. Ultrasonography of the abdomen shows a nodular liver. Which of the following etiopathogenetic mechanisms is most likely associated with the development of this condition? A. Repeated blood transfusion B. Increased iron absorption in the gut C. Obstruction of the extra-hepatic bile ducts D. Impaired copper excretion in bile E. Increased lysis of red blood cells

D. Impaired copper excretion in bile

2. A 21-year-old woman comes to physician because of a 2-week history of fatigue, nausea and vomiting. She denies fevers, abdominal pain and recent travel. However, a coworker contracted meningitis 2 months ago and she received chemoprophylaxis with rifampin. Her only medication is a combined oral contraceptive for the past 3 years. Her vitals are within normal limits and physical examination shows no abnormalities except breast tenderness. A urine pregnancy test is positive. Which of the following best explains the contraceptive's failure despite the patient's compliance with the medication? A. Inhibition of hepatic P450 enzymes by rifampin B. Inhibition of rifampin by hepatic P450 enzymes C. Induction of rifampin by hepatic P450 enzymes D. Induction of hepatic P450 enzymes by rifampin E. Induction of oral contraceptive by hepatic P450 enzymes

D. Induction of hepatic P450 enzymes by rifampin

A 40-year-old man comes to the physician because of a 2-month history of a white patch on his tongue. Physical examination shows a well-defined white patch on the right lateral border of his tongue. Laboratory studies show low CD4 count which is suggestive of immunodeficiency. Histological examination of a biopsy specimen taken from the lesion shows hyperkeratosis and acanthosis. Which of the following risk factors is most likely associated with this patient's condition? A. Poorly fitting Dentures B. Infection with Candida albicans C. Chewing Tobacco D. Infection with Epstein-Barr virus E. Infection with Human papilloma virus

D. Infection with Epstein-Barr virus

A 71-year-old man presents with dysphagia and is found to have a 5- cm mass that is located in the middle third of the esophagus and extends into adjacent lung tissue. Which of the following statements best describes the expected microscopic appearance of this lesion? A. A mass composed of benign cartilage B. A mass composed of benign smooth-muscle cells C. Infiltrating groups of cells forming glandular structures D. Infiltrating sheets of cells forming keratin E. Infiltrating single cells having intracellular mucin

D. Infiltrating sheets of cells forming keratin SCC = nest/islands of cells, keratinization, keratin pearls

A 38-year-old businessman comes to the physician for a preventative visit in advance of an extended trip through many South American countries. In addition to vaccines and malaria prophylaxis he is advised to carry the anti-diarrheal drug diphenoxylate with him. Which of the following best describes the mechanism of action of this drug? A. Inhibits serotonin receptors B. Antihistaminergic C. Broad-spectrum antibiotic D. Inhibits ACh release from enteric neurons E. Sympathetic nervous system stimulant

D. Inhibits ACh release from enteric neurons Loperamide - not as addictive Diphenoxylate - crosses BBB so can be more addictive

A 25-year-old man comes to the physician for recurring episodes of bloody diarrhea and severe abdominal pain. A colonoscopy shows rectal and sigmoidal pseudopolyps with erythematous, friable mucosa. He is prescribed a therapeutic enema. Which of the following best describes the mechanism of action of the drug likely to be given via this enema? A. Antimicrobial effects B. Blocks TNF alpha C. Inhibits dihydrofolate reductase D. Inhibits phospholipase A2 E. Slows peristalsis

D. Inhibits phospholipase A2

***A 3-year-old boy is brought to the physician by his mother because of a 3-day history of rectal bleeding. His vital signs are within normal limits. Physical examination shows no other abnormalities. Colonoscopy shows a 2-cm, pedunculated, reddish mass in the rectum. Biopsy from the lesion shows dilated glands filled with mucin and, expanded lamina propria with inflammatory debris. Which of the following is the most likely diagnosis? A. Peutz-Jeghers syndrome B. Lynch syndrome C. Familial adenomatous polyposis D. Juvenile polyp E. Diverticulosis

D. Juvenile polyp

A 23-year-old woman at 7 months' gestation comes to the physician for a prenatal examination. She has a history of mild hyperthyroidism. Despite her advanced pregnancy, she is found to have lost 2 kg (4.4 lb) since her last visit a month ago. She also has developed a fine tremor of her fingers. Her pulse is 120/min, and her thyroid gland is noticeably larger than it was at the time of her last office visit. Which of the following is the most appropriate pharmacotherapy for this patient? A. Levothyroxine B. Radioactive iodine C. Iodide D. Methimazole E. Propylthiouracil

D. Methimazole

8. A 26-year-old man comes to the physician because of a 3-day history of fever and penile discharge. He had an unprotected sexual encounter with a woman 1-week prior to the onset of his symptoms. His temperature is 38.2°C (100.6°F), respirations are 16/min, pulse is 80/min and blood pressure is 121/75 mm Hg. Physical examination shows a purulent penile discharge. Laboratory investigations of a swab sample of the discharge shows Gram negative, intracellular, diplococci. Which of the following agars will most likely be used to culture the causal agent? A. MacConkey Agar B. Diamond's media C. Salmonella-Shigella agar D. Modified Thayer-Martin agar E. Lowenstein Jensen agar

D. Modified Thayer-Martin agar

A 20-year-old woman comes to the physician for a follow-up. She has a history of type 1 diabetes mellitus with severely compromised beta cell function and requires insulin therapy for glycemic control. A trial using twice daily pre-meal subcutaneous injections of an intermediate-acting insulin is started. Which of the following forms of insulin will this patient be taking? A. Aspart B. Detemir C. Glargine D. NPH E. Degludec

D. NPH

A 35-year-old woman comes to the physician for a scheduled cervical screening. She has regular menstrual cycles, and her last menstrual period was 2 weeks ago. A pelvic examination shows a small polypoid mass at the cervical os. A biopsy from the lesion shows a benign neoplastic cervical polyp composed of mucin-producing columnar epithelial cells with a fibrous stroma. Which of the following is the likely course of this disease? A. Spontaneous regression B. Conversion to malignancy C. Cyclical growth D. No malignant potential E. Recurrence after resection

D. No malignant potential

A 69-year-old man comes to the physician because of a 5-week history of abdominal pain, nausea, vomiting, and non-bloody diarrhea. He was diagnosed with HIV 8-years ago and is non-compliant with antiretroviral therapy. Laboratory studies show a CD4 cell count of 90 cells/mm3 and an HIV viral load of 11, 200 copies/mL. A modified acid-fast staining of his stool sample shows the presence of several oocysts measuring approximately 4-6 µm (see side image). Which of the following is most likely the causal pathogen? A. GI invasive virus B. Gram negative bacteria C. Flagellate protozoan D. Non-motile protozoan E. Gram positive bacteria

D. Non-motile protozoan

A 46-year-old woman comes to the physician because of a 3-day history of epigastric pain and difficulty swallowing which has made it difficult to eat. She has a medical history of being HIV positive, diagnosed 4 years ago; last CD4+ cell count was 200. An upper endoscopy shows patchy white plaques on her esophagus. A cytology brush is used to collect a sample, that is sent to the laboratory for culture and microscopic investigation. Which of the following is the most likely source of infection? A. Contaminated food B. Contaminated water C. Infected person D. Normal flora E. Reactivation of a latent infection

D. Normal flora

A 29-year-old woman comes to the physician because of abdominal pain and diarrhea. Endoscopy shows the presence of peptic ulcers. A diagnosis of Zollinger-Ellison syndrome (gastrin-secreting neuroendocrine tumor) is made. Which of the following treatments would be most effective in reducing this patient's symptoms? A. Magnesium hydroxide B. Misoprostol C. Ranitidine D. Omeprazole E. Sucralfate

D. Omeprazole

A 58-year-old woman comes to the physician because of heartburn and abdominal pain for six months. She has been using over the counter preparations without complete resolution of her symptoms. Imaging studies show a small gastrinoma. Which of the following is the most appropriate pharmacotherapy for this patient. A. Cimetidine B. Sucralfate C. Fexofenadine D. Omeprazole E. Bismuth Subsalicylate

D. Omeprazole

A 35-year old previously healthy woman comes to the physician because of palpitations and unintentional weight loss for the past 5 months. She does not have a history of heart problems and is losing weight despite her unchanged diet. Her blood pressure is 140/80 mm Hg, pulse is 120/min, respirations are 22/min, and temperature is 37C (98.6F). She does not take any medications and denies any illicit drug use. Physical examination shows moist and wet skin, thinning hair and absence of a neck mass. Ultrasound of the thyroid shows no abnormalities and radioiodine studies show decreased uptake in the thyroid gland. Laboratory studies show: T4 - 15µg/dL (5-12µg/dL); TSH - 0.4µU/mL (0.5-5.0µU/mL); Cardiac markers - negative An ultrasound of which of the following organs is most likely to confirm the diagnosis? A. Breast B. Thyroid C. Parathyroid D. Ovary E. Uterus

D. Ovary; Ultrasound of the thyroid shows no abnormalities and radioiodine studies show decreased uptake in the thyroid gland. So it's an ectopic production of thyroid hormone = struma ovariii (Know!)

3. A 28-year-old woman, G3P2 at 38 weeks comes to the emergency department because of 4-hour history of progressively worsening abdominal and lower back pain. Her blood pressure is 128/80 mmHg, pulse is 110/min, respirations are 20/min and temperature is 37oC (98.6 F). Physical examination shows a woman in painful distress with an unfavorable cervix. Which of the following drugs is most likely contraindicated in this patient? A. Misoprostol B. Dinoprostone C. Magnesium sulphate D. Oxytocin E. Methylergonovine

D. Oxytocin Misoprostol would be used to ripen the cervix in an UNFAVORABLE cervix; oxytocin is contraindicated because you don't want to start contractions with an unfavorable cervix

2. A 46-year-old woman comes to the physician because of a 4-month history of nipple discharge and itching over the areola. She also complain of weakness and tiredness. Physical examination shows an erythema and scaly crust over the areola. An excisional biopsy of the nipple area shows infiltration of the nipple by large cells with hyperchromatic nucleus clear cytoplasm. These cells are found both singly and in small clusters in the epidermis. What of the following is the most likely diagnosis? A. Ductal papilloma B. Eczematous inflammation C. Mammary duct ectasia D. Paget's disease E. Phyllodes tumor, malignant

D. Paget's disease

A 22-year-old woman with 4 months of amenorrhea comes to the emergency department because of 2-hour history of vaginal bleeding. Urine pregnancy test is positive. Her temperature is 36.9C (98.4F), blood pressure is 116/72 mmHg, pulse is 80/m, respirations are 20/m. Physical examination shows minimally enlarged uterus. Ultrasonography of the abdomen and pelvic shows an intrauterine gestational sac without cardiac activity. Dilatation and curettage was performed. Examination of specimen obtained during D&C shows focal trophoblastic hyperplastic cells and enlarged villi mixed with normal villi. What is the most likely diagnosis? A. Complete mole B. Invasive mole C. Choriocarcinoma D. Partial mole E. Multiple pregnency

D. Partial mole

*A 44-year-old man comes to the emergency department because of an accidental needle-stick injury while drawing blood from a patient 3-days ago. He is concerned that he may develop viral hepatitis. His vital signs and physical examination are within normal limits. Laboratory studies of his serology tests are shown (see below). Which of the following best explains his hepatitis status? A. Co-infection with hepatitis A and B B. Past hepatitis A and B infection C. Past hepatitis A infection and current chronic hepatitis B D. Past hepatitis A infection and vaccinated against hepatitis B E. Current hepatitis A infection and resolved hepatitis B

D. Past hepatitis A infection and vaccinated against hepatitis B

A 55-year-old man comes to the physician for a routine follow-up examination. He has a 15-year history of type II diabetes mellitus and is currently being managed with metformin. He drinks alcohol socially and exercises once a week. His BMI is 31 kg/m2. Physical examination shows no abnormalities. Laboratory studies show fasting blood glucose 138 mg/dL (Normal: 70-110 mg/dL), blood urea nitrogen 18 mg/dL (Normal: 7 - 18 mg/dL) and serum creatinine 1.1 mg/dL (Normal: 0.6 - 1.2 mg/dL). Urinalysis shows 1+ proteinuria. Which of the following long-term complications is most likely to develop in this patient? A. Ketoacidosis B. Nephritic syndrome C. Acute interstitial nephritis D. Peripheral neuropathy E. Subarachnoid hemorrhage

D. Peripheral neuropathy, nephropathy and retinopathy —-> advanced glycosylation of end products Not B or C because diabetes nephropathy is a NEPHROTIC syndrome

A 35-year-old man comes to the emergency department because of a 1-hour history of nausea and severe vomiting at a family picnic 4 hours ago. The food at the picnic included baked chicken, coleslaw and macaroni pie. His condition improves after a few hours in the emergency room. Which of the following most likely caused his symptoms? A. Parasitic infection B. Preformed endotoxin in food C. Invasion of the gut by bacteria D. Preformed exotoxin in food E. Activation of systemic immune response

D. Preformed exotoxin in food Most of your food poisoning bacteria are gram positive (Neisseria is LOS; LPS is for every other gram negative organism)

A 36-year-old woman presents to the urgent care center with lightheadedness and fatigue for 2 weeks. She denies fever, chills, and pain. BP is 90/60 mmHg. Her conjunctivae are pale and capillary refill is >2 seconds. Her hemoglobin is 8.9 g/dL (normal: 12.0-16.0 g/dL). Which of the following best helps to confirm the diagnosis? A. Rebound tenderness B. Inguinal examination C. Abdominal CT scan D. Rectal examination E. Abdominal ultrasound

D. Rectal examination

A 35-year-old man come to the Emergency department because of a 7-day history of abdominal pain, fever and bloody diarrhea. This is his second time experiencing these symptoms in 6 months. He was diagnosed with GERD 2 years ago managed on proton pump inhibitors (PPI's). He is a poultry farmer. His T is 36.6oC (98oF), PR 92/min, RR is 20/min. Physical examination shows generalized abdominal tenderness. Laboratory studies reveals a Gram negative, non-lactose fermenting, H2S producing rod. Which of the following is the most likely causal agent causing this man's symptoms? A. Shigella dysenteriae B. Shiga Toxin Escherichia coli C. Enterohemorrhagic Escherichia coli D. Salmonella Enteritidis E. Vibrio vulnificus

D. Salmonella Enteritidis

1. A 60-year-old man comes to the physician because of recurrent heart burn for the past 2 months. Physical examination shows no apparent abnormalities. An upper gastrointestinal endoscopy is performed and a biopsy from the mucosa of lower one-third of the esophagus is obtained. Which of the following histological findings would most likely suggest a normal esophageal mucosa? A. Glandular epithelium B. Pseudostratified columnar epithelium C. Ciliated columnar epithelium D. Squamous epithelium E. Transitional epithelium

D. Squamous epithelium

A 46-year-old woman comes to the physician because of a 3-week history of progressively worsening palpitations and dizziness. Her pulse is 130/min blood pressure is 143/88 mm Hg and respirations are 26/min. Physical examination shows an anxious man. An ECG shows an irregularly irregular pattern with varying R-R intervals and no p waves. The patient is prescribed a drug which blocks L-type calcium channels in the heart. The physician warns the patient about a common adverse effect of this drug and recommends an over-the-counter drug if the patient experiences this adverse effect. Which of the following best describes the mechanism of action of this over-the-counter drug? A. Neutralization of gastric acid B. 5HT3 receptor antagonism C. Inhibition of ACh release D. Stimulant laxative E. 5HT4 receptor agonism

D. Stimulant laxative

A 28 year old male presents to the ED with complaints of a 3-day history of abdominal pain. The pain is aching and is localized to the epigastrium. Symptoms started after he took pain killers(Ibuprofen) prescribed by his Physician for a knee injury. He has associated nausea, dark vomitus and the pain is worse after meals.From which of the following organs is the pain likely originating. A. Liver B. Spleen C. Colon D. Stomach E. Pancreas

D. Stomach

9. A 68-year-old woman is admitted to hospital because of a 2-day history of abdominal cramps, low-grade fever, and 4-7 watery bowel movements in the last 6 hours. She recently completed a course of antibiotics for treatment of pneumonia due to Pseudomonas aeruginosa. Her pulse is 121/min, respirations are 18/min and blood pressure is 100/60 mm Hg. She is suspected of having antibiotics associated colitis. Which of the following stool analyses is most useful in diagnosing this patient? A. Routine culture B. Gram stain of stool C. Stool test for fecal leukocytes D. Stool test for Toxin A and B E. Stool test for Ova and Parasites

D. Stool test for Toxin A and B

**A 12-year-old girl is admitted to hospital because of a 6-hour history of abdominal cramps, low-grade fever, and 4 watery bowel movements in the last 6 hours. She has cystic fibrosis and recently completed a course of antibiotics for treatment of pneumonia due to Pseudomonas aeruginosa. Her pulse is 121/min, respirations are 18/min and blood pressure is 100/60 mm Hg. She is suspected of having antibiotics associated colitis. Which of the following diagnostic tests is most likely to be ordered to confirm a diagnosis in this patient? A. Endoscopy and biopsy B. Gram stain of stool C. Stool test for fecal leukocytes D. Stool test for tcdA and tcdB E. Stool test for Ova and Parasites

D. Stool test for tcdA and tcdB

A 5-year-old girl is brought to the physician by her parents because of severe weight loss. Her parents are concerned as over the past 3 months, their daughter has lost 8 kg and has "not been herself". They describe their daughter as being lethargic despite eating and drinking an increased amount. Her parents have also noticed that she has been urinating more frequently. Vitals are as follows: pulse 110 BPM, blood pressure of 100/70 mmHg and respiration were 20/minute. A random blood glucose shows a blood sugar level of 201 mg/dL. Which of the following immunological responses is most likely the cause of this condition? A. Type I hypersensitivity reaction B. Type II hypersensitivity reaction C. Type III hypersensitivity reaction D. Type IV hypersensitivity reaction E. Peripheral Insulin Resistance

D. Type IV hypersensitivity reaction

A 74-year-old woman comes to the physician because of a 5-day history of worsening bloating, nausea, weight loss and a dull pain in her stomach. The pain is worse about 4 hours after eating and wakes her up at night. A non-invasive breath test is positive. Which of the following virulence factors is most likely used by the causal agent to modify the pH microenvironment of the stomach? A. Adhesin B. Flagella C. Oxidase D. Urease E. CagA toxin

D. Urease

8. A 70-year-old man comes to the physician because of a 3-month history of progressive visual difficulties. He says that he is seeing dark spots and has difficulty seeing in dim light. He has a 30-year history of type-2 diabetes mellitus, for which he is taking oral medications. Fundoscopic examination of the eye shows microvascular lesions in the retina. Laboratory studies show serum glucose of 160 mg/dL and HbA1C of 8.5%. Which of the following best explains the pathogenesis of the retinal lesions in this patient? A. CMV infection of the retina B. Granulomatous inflammation involving retinal arteries C. Increased ketone body production D. Neuropathic damage to the optic nerve E. Advanced glycosylation end products

E. Advanced glycosylation end products

A 60-year-old man comes to the physician because of a 2-month history of progressive visual difficulties. He says that he is seeing dark spots and has difficulty seeing in dim light. He has a 20-year history of type-2 diabetes mellitus, for which he is taking oral medications. Fundoscopic examination of the eye shows microvascular lesions in the retina. Laboratory studies show serum glucose of 160 mg/dL and HbA1C of 8.5%. Which of the following best explains the pathogenesis of the retinal lesions in this patient? A. CMV infection of the retina B. Granulomatous inflammation involving retinal arteries C. Increased ketone body production D. Neuropathic damage to the optic nerve E. Advanced glycosylation end products

E. Advanced glycosylation end products

A 36-year-old woman comes to the physician because of a 1-month history of frequent episodes of epigastric pain which are relieved by eating. Abdominal examination shows no abnormalities. She is scheduled for upper GI endoscopy the following week. She is prescribed a medication which reduces basal secretion of gastric acid. Which of the following is most likely mechanism of action of this drug? A. Blockade of gastrin receptors B. Inhibition of synthesis of gastrin C. Inhibition of H+/K+ ATPase activity D. Prostaglandin-mediated inhibition of acid secretion E. Antagonism of H2 receptors

E. Antagonism of H2 receptors

A 30-year-old woman comes to the emergency department because of a 2-hour history of nausea and 6 episodes of vomiting not associated with diarrhea or fever. There is no blurred vision or pruritis. She ate a bowl of leftover beef and lentil soup 4 hours ago. Her temperature is 37.2oC (98.9oF), pulse is 90/min and blood pressure is 100/80mm Hg. Which of the following toxins is the most likely cause of her symptoms? A. ETEC toxin B. EPEC toxin C. Vibrio cholerae toxin D. Clostridium perfringens toxin E. Bacillus cereus toxin

E. Bacillus cereus toxin - Only one associated with food poisoning

A 24-year-old man comes to the physician because of a 1-day history of abdominal pain and watery diarrhea. He denies fever, nausea or vomiting. He is currently on vacation in Australia with his family. All of his family members have similar symptoms. Their symptoms started 18 hours after eating Asian food from a buffet at a local restaurant. Laboratory studies show that one of the dishes served at the restaurant contains >107 anaerobic gram-positive bacteria/g of food. Which of the following is the most likely causal agent for their symptoms? A. B. cereus B. Enteropathogenic E.coli C. Enterotoxigenic E. coli D. V. cholerae E. C. perfringens

E. C. perfringens

A 56-year-old woman comes to her physician because of vaginal bleeding for the past 4 weeks. Her last menstrual period was six years ago. She also reports progressive fatigue, loss of appetite and unintentional weight loss of 3kg over a period of 1 month. Vital signs are within normal limits. A pelvic ultrasound shows a thickened endometrium and a mass on the right ovary. Which of the following is most likely seen on a biopsy of the mass? A. Tissues derived from all 3 germ layers B. Reinke crystalloids C. Large, uniform round cells with lymphocytic invasion D. Signet ring cells E. Call-exner bodies

E. Call-exner bodies (granulosa ovarian tumor)

Case5: 38 year old G6P6 LMP 1 week ago complains of post coital bleeding and heavier periods for one year. Her last Pap smear was 5 years ago. She never received results . She denies other bleeding problems , now has malodourous vaginal discharge On speculum examination , there is, and a friable grayish red lesion replacing her cervix. The uterus is normal size , no adnexal mass / tenderness. Rectovaginal examination : thickening of the parametriumWhat is the most likely diagnosis? A. Humanpapillomavius 11 B. Humanpapillomavirus 16 C. Moderate cervical dysplasia D. Cervical Polyp E. Cervix Cancer

E. Cervix Cancer

A 25-year-old woman comes to the physician for follow up examination. She is a commercial sex worker. About 1-year ago, she was diagnosed with acute Hepatitis B infection. Her temperature is 39°C (102.2°F), pulse is 130/min, respirations are 20/min and blood pressure is 120/76 mmHg. Physical examination shows scleral icterus and right subcostal tenderness. Her current serology report is shown. Which of the following best describes her current infection? A. Acute Hepatitis B B. Currently recovering from Hepatitis B infection C. Past Hepatitis B infection D. Vaccinated against Hepatitis B E. Chronic Hepatitis B

E. Chronic Hepatitis B

A 48-year-old woman comes to the physician because of a 2 month history of regurgitation of partially digested food and occasional nausea and vomiting. She is treated with a drug that prevents nausea and vomiting and promotes upper gastrointestinal motility. Blockade of which of the following receptors is partially responsible for the therapeutic effects of this drug? A. M2 cholinergic B. Nn cholinergic C. Beta-2 adrenergic D. H2 histaminergic E. D2 dopaminergic

E. D2 dopaminergic

A 48-year-old woman comes to the physician because of a 2-month history of regurgitation of partially digested food and occasional nausea and vomiting. She is treated with a drug that prevents nausea and vomiting and promotes upper gastrointestinal motility. Blockade of which of the following receptors is partially responsible for the therapeutic effects of this drug? A. M2 cholinergic B. Nn cholinergic C. Beta-2 adrenergic D. H2 histaminergic E. D2 dopaminergic

E. D2 dopaminergic

A 68-year-old man comes to the physician because of nocturia, hesitancy, urgency, urinary dribbling and weak urinary stream. Ultrasound shows enlarged prostate. He is started on finasteride. After 4 months on this treatment, his symptoms improve markedly, and his prostate has regressed in size. Which of the following mechanisms most likely mediates this benefit? A. Inhibition of gonadal steroid synthesis B. Inhibition of testosterone binding to receptors C. Increase conversion of testosterone to DHT D. Increase conversion of DHT to testosterone E. Decrease conversion of testosterone to DHT

E. Decrease conversion of testosterone to DHT

A 44-year-old woman comes to the physician because of severe pelvic pain during both menstrual and non-menstrual days. The patient was diagnosed with endometriosis one year earlier. She had good pain relief after surgical ablation of endometriotic lesions, but her symptoms have returned. She is prescribed a cycle of continuous therapy with leuprolide. Which of the following adverse effects would be expected to occur during the therapy? A. Galactorrhea B. Vaginal Infections C. Menorrhagia D. Ovarian hyperstimulation E. Diminished libido

E. Diminished libido

A 10-year-old boy is diagnosed with type 1 diabetes and started on insulin therapy. Which of the following insulin regimens would be most appropriate for the chronic therapy of this patient? A. A B. B C. C D. D E. E

E. E

3. A 37-year-old woman comes to the physician after noticing a lump in the right breast. Physical examination of the right breast shows a small, mobile, well circumscribed mass. Mammography shows a calcification. She recently had a renal transplant and is put on some medication which she doesn't remember. Which of the following is the most likely diagnosis of this patient. A. Acute mastitis B. Duct Ectasia C. Intraductal Papilloma D. Phyllodes Tumor E. Fibroadenoma

E. Fibroadenoma

4. A 21-year-old woman comes to the physician because of a 1-week history of pelvic pain. She is sexually active and has had multiple sexual partners. Her temperature is 37°C (98.6°F), pulse is 86/min, respirations are 18/min and blood pressure is 118/70 mm Hg. Pelvic examination shows mild erythema of the ectocervix. Pap smear shows numerous neutrophils, but no dysplastic cells. A culture of a cervical smear grows Neisseria gonorrhoeae. Which of the following complications is most likely to develop, if the infection is not treated appropriately? A. A. Cervical adenocarcinoma B. B. Sarcoma botryoides C. C. Endometrial hyperplasia D. D. Endometriosis E. E. Ectopic pregnancy

E. E. Ectopic pregnancy

5. A 45-year-old woman comes to the physician because of a 1-month history of a small amount of vaginal bleeding and a brownish, foul-smelling discharge. She has a history of HPV 18 infection. Physical examination of the pelvis shows a 3-cm lesion on the ectocervix which is an exophytic, polypoidal, fungating mass which projects into the vagina. Inguinal lymph nodes are also palpable. Which of the following is the most likely diagnosis? A. A. Adenocarcinoma B. B. Cervical intraepithelial neoplasia C. C. Chronic cervicitis D. D. Extramammary Paget disease E. E. Squamous cell carcinoma

E. E. Squamous cell carcinoma

10. A 25-year-old woman comes to the physician because of a 1-day history of abdominal pain and watery diarrhea. She says she ate at a local restaurant 48 hours ago. Another 8 of her friends have similar symptoms. Laboratory studies show ≥106 C. perfringens/g stool. Further investigations identified corned beef as the source of the outbreak. The beef had been cooked 4 days in advance and stored in a refrigerator. Approximately 90-minutes before serving, the meat was sliced and placed under heat lamps. Which of the following characteristics is most likely to be involved in the pathogenesis of the causal agent? A. Anaerobic B. Endotoxin C. Invasive D. Motility E. Endospores

E. Endospores

A 45-year-old man comes to the physician because of a 4-week history of abdominal pain, fever and malaise. He is homeless person with a history of intravenous drug use and multiple unprotected sexual encounters both with men and women. His temperature is 38.8°C (101.8°F), pulse is 101/min and blood pressure is 120/80 mmHg. Physical examination shows hepatomegaly and icterus. Laboratory studies show elevated AST, ALT levels and presence of HBsAg. He is diagnosed with Hepatitis B. Which of the following features is most likely associated with the causal pathogen? A. Non-enveloped, positive sense, single stranded RNA B. Enveloped, negative sense, segmented RNA C. Enveloped, negative sense, non-segmented RNA D. Enveloped, positive sense, single stranded RNA E. Enveloped, partially double stranded DNA

E. Enveloped, partially double stranded DNA

A 45-year-old man comes to the physician because of a 4-week history of abdominal pain, fever and malaise. He is homeless person with a history of intravenous drug use and multiple unprotected sexual encounters both with men and women. His temperature is 38.8°C (101.8°F), pulse is 101/min and blood pressure is 120/80 mmHg. Physical examination shows hepatomegaly and icterus. Laboratory studies show elevated AST, ALT levels and presence of HBsAg. He is diagnosed with Hepatitis B. Which of the following features is most likely associated with the causal pathogen? A. Non-enveloped, positive sense, single stranded RNA B. Enveloped, negative sense, segmented RNA C. Enveloped, negative sense, non-segmented RNA D. Enveloped, positive sense, single stranded RNA E. Enveloped, partially double stranded DNA

E. Enveloped, partially double stranded DNA A- hep a B - influenza D - hep c

A 58-year-old obese woman comes to the physician for a follow-up examination. She was diagnosed with type 2 diabetes mellitus 9 months ago and started on metformin. Metformin was progressively titrated up to maximal dose. Despite therapy with high-dose metformin her hemoglobin A1c is still 8.4%. A more potent second antidiabetic agent is added to her therapeutic regimen. The physician anticipates that her weight will decrease but warns the patient that this drug can cause acute pancreatitis. Which of the following antidiabetic agents was this patient most likely prescribed? A. Glipizide B. Rosiglitazone C. Nateglinide D. Acarbose E. Exenatide

E. Exenatide

An 18-year old man comes to the physician because of a 3-day history of reduced appetite, abdominal cramps, bloating, flatulence, and foul-smelling, non-bloody, greasy stools (3 - 4 episodes per day). About 10 days ago, he returned from a summer camp in the Blue Ridge Mountains, where they used a mountain stream as their main water supply. His temperature is 38°C (100.4°F), pulse is 104 /min and blood pressure is 115/72 mmHg. Physical examination shows slightly dry mucous membranes and hyperactive bowel sounds. Which of the following parasitic agents is most likely to be observed on an O & P examination of his stool sample? A. Sporulated oocysts B. Sporozoite and oocysts C. Unsporulated oocysts D. Ciliated trophozoites and cysts E. Flagellate trophozoites and cysts

E. Flagellate trophozoites and cysts

A 50-year-old man comes to the physician because of a 4-month history of difficulty swallowing and epigastric pain. He initially had difficulty in swallowing solids but now cannot swallow liquids either. He has a 9-kg (19.8-lbs) weight loss during this period. His temperature is 37°C (98.6°F), blood pressure is 110/80 mm Hg, pulse is 70/min and respirations are 20/min. An upper GI endoscopy shows an exophytic mass arising from the lower third of the esophagus. Microscopic examination of a biopsy specimen obtained from the mass shows malignant cells forming glands and infiltrating the underlying stroma. Which of the following is the most likely risk factor for the development of this condition? A. Cigarette smoking B. Iron deficiency C. Vitamin A deficiency D. Gluten hypersensitivity E. Gastroesophageal reflux

E. Gastroesophageal reflux In terms of most significant RF: Barrett's Esophagus, GERD, smoking

A 33-year-old man comes to the physician because of a 2-week history of abdominal pain, fever and malaise. He returned 3-months ago from a vacation to Costa Rica, South America. While on vacation, he frequently consumed ethnic foods from local roadside eateries and had multiple unprotected sexual encounters. His temperature is 38°C (100.4°F), pulse is 90/min and blood pressure is 110/70 mmHg. Physical examination shows hepatomegaly and mild icterus. Laboratory studies show elevated AST and ALT levels. RT-PCR analysis identifies a virus harboring an RNA genome surrounded by an envelope. Which of the following is most likely the causal pathogen? A. Coronavirus B. Norovirus C. Influenza virus D. Hepatitis A virus E. Hepatitis C virus

E. Hepatitis C virus

3. A 65-year-old man is brought to the emergency department because of sudden onset of hematemesis. He drinks 100ml of alcohol per day for the last 20 years. His temperature is 37.5°C (99.5°F), pulse is 80/min, respirations are 25/min and blood pressure is 90/60 mm Hg. Physical examination shows distended abdomen and bilateral pitting pedal edema. An ultrasound of the abdomen shows a shrunken liver. This patient is at risk of developing which of the following complications? A. Splenic infarction B. Fibrosis of right side of the heart C. Boerhaave syndrome D. Zollinger-Ellison syndrome E. Hepatocellular carcinoma

E. Hepatocellular carcinoma

2. A 75-year-old man is brought to the emergency department after he was found in an unconscious state on the roadside. His temperature is 37.5°C (99.5°F), pulse is 80/min, respirations are 25/min and blood pressure is 110/70 mm Hg. His breath smells of alcohol. Physical examination shows mild to moderate respiratory distress, distended abdomen and bilateral pitting pedal edema. An ultrasound of the abdomen shows hepatomegaly. Which of the following is the most likely serum finding in this condition? A. Increased AST/ ALT <1 B. Increased 5-HT C. Increased ferritin D. Increased serum albumin E. Increased AST/ALT >2

E. Increased AST/ALT >2

2. A 75-year-old man is brought to the emergency department after he was found in an unconscious state on the roadside. His temperature is 37.5°C (99.5°F), pulse is 80/min, respirations are 25/min and blood pressure is 110/70 mm Hg. His breath smells of alcohol. Physical examination shows mild to moderate respiratory distress, distended abdomen and bilateral pitting pedal edema. An ultrasound of the abdomen shows hepatomegaly. Which of the following is the most likely serum finding in this condition? A. Increased cerulopasmin B. Increased 5-hydroxytryptamine C. Increased ferritin D. Increased serum albumin E. Increased aminotransferases

E. Increased aminotransferases

1. A 60-year-old woman comes to the physician because of lethargy, altered bowel habits and recurrent bloody stools for the past 3 months. Her blood pressure is 120/70 mmHg, pulse is 115/min, respiration are 13/min and temperature is 37.090C(98.70F). Physical examination shows conjunctival pallor and tenderness over the left lower quadrant. Imaging studies show presence of mucosal outpouching from the colonic wall along the mesenteric border. Which of the following is a likely risk factor for this condition? A. Systemic atherosclerosis B. HLA-DRB1 association C. Tropheryma whippelii infection D. Antibiotic therapy E. Lack of fiber in the diet

E. Lack of fiber in the diet

A 38-year-old man is diagnosed with type 2 diabetes mellitus. He has a history of hyperlipidemia, well controlled with rosuvastatin, and hypertension, well controlled with metolazone and lisinopril. He is prescribed the first line drug for diabetes type 2. Which of the following adverse effects may he experience? A. Edema B. Hypoglycemia C. Diarrhea and Flatulence D. Weight Gain E. Lactic Acidosis

E. Lactic Acidosis

2. A 40-year-old woman comes to the physician because of a 2-week history of fatigue, itching in her ankles and right upper quadrant discomfort. She consumes 1 glass of wine occasionally. Physical examination shows mild icterus, hyperpigmentation of the skin and xanthelasmas. There is no evidence of ascites, subcutaneous bleeds, spider nevi or flapping tremor. Liver function tests show increased serum bilirubin, alkaline phosphatase, alanine and aspartate aminotransferase. Serum studies show anti-mitochondrial antibodies. Which of the following histologic changes is most likely to be seen in this patient's liver? A. Inflammation and fibrosis of the intra and extrahepatic biliary tree B. Distorted architecture, regenerative nodules with bridging fibrosis C. Ballooning of hepatocytes with lipid accumulation D. Steatohepatitis with mallory hyaline and fibrosis around central vein E. Non suppurative, granulomatous destruction of intrahepatic bile ducts

E. Non suppurative, granulomatous destruction of intrahepatic bile ducts

2. A 55-year-old woman come to the physician because of a 1-week history of frequent passage of foul-smelling mucoid stool, crampy abdominal pain, tenesmus and fever for the past 2 days. Her stool assay is positive for clostridium difficile toxin. Colonoscopy demonstrates a raised yellow plaques consisting of fibrinopurulent necrotic debris. Which of the following is the most likely factor leading to her condition? A. Family history of Colon carcinoma B. Consumption of unpasteurized milk C. Recent angiographic procedure D. History of recent travel outside US E. Recent use of broad-spectrum antibiotics

E. Recent use of broad-spectrum antibiotics

A 3-year-old boy is brought to the physician by his mother because of a 2-day history of vomiting and watery stools. She denies any blood or mucus in the stool. The family returned from a vacation in the UK 3-days ago. His vaccinations are up to date. His temperature is 37.7°C (99.9°F), pulse is 140/min, respirations are 24/min, and blood pressure is 80/60 mmHg. Physical exam shows a soft, non-tender, abdomen with hyperactive bowel sounds. Which of the following best describes the causal agent? A. Ds DNA virus B. Thick-walled oocyst C. Gram negative bacillus D. Gram positive bacillus E. Ss RNA virus

E. Ss RNA virus

3. A 50-year-old woman comes to the physician because of a lump in her right breast. She first noticed the lump 2-months ago and says that it has been progressively increasing in size. Physical examination shows a deep, hard, nodular mass in the upper outer quadrant of the right breast, which is immobile on palpation. Palpation also shows non-tender enlargement of axillary lymph nodes. A working diagnosis of carcinoma of the breast is made. Which of the following is the best laboratory study to confirm this diagnosis at this point? A. Ultrasonography of the breast B. MRI of the breast C. Mammography D. Blood Examination E. Tissue biopsy

E. Tissue biopsy

A 36-year-old woman comes to the emergency department because of dyspnea on exertion, tremor, and heat intolerance. Her blood pressure is 167/95 mmHg, and pulse is 130/min. Physical examination shows an enlarged thyroid gland. Laboratory studies confirm the diagnosis of Graves' disease. She is started on methimazole and propranolol. Two months later the patient's symptoms have not improved, and her blood thyroid hormone levels remain elevated. She is scheduled for thyroidectomy. Seven days before the procedure she is started on iodide solution. Which of the following best explains why this drug is given before thyroid surgery? A. To prevent conversion of peripheral T4 to T3. B. To inhibit thyroid hormone synthesis. C. To reduce thyrotoxic symptoms. D. To reduce adverse effects of thioamides E. To decrease vascularity of the thyroid gland.

E. To decrease vascularity of the thyroid gland

2. A 27-year-old woman who is a marathon player comes to the physician because of a painful lump in her upper outer quadrant of right breast. A mammogram shows irregular mass with focal area of calcification. An excisional biopsy shows a few macrophage, proliferating fibroblast and a localizes area of granulation tissue with chalky white foci. Which of the following is the most likely diagnosis? A. Acute mastitis B. Ectasia C. Enzymatic fat necrosis D. Foreign-body reaction E. Traumatic fat necrosis

E. Traumatic fat necrosis

11. A 25-year-old woman comes to the physician because of a 2-day history of fever and an unusual smelling vaginal discharge. She is sexually active with her new boyfriend who recently migrated from the Caribbean. Her temperature is 38.2°C (100.8°F), respirations are 16/min, pulse is 75/min and blood pressure is 125/75 mm Hg. Physical examination shows a yellowish-greenish, malodorous vaginal discharge and cervical petechiae. Laboratory studies of the discharge show motile trichomonads and a positive Whiff test. Which of the following best describes the pathogenesis of the causal agent? A. Adhesion via opacity proteins B. Biphasic life cycle C. Lipophosphoglycan (LPG) attachment D. Giant cell formation E. Trophozoite to amoeba transformation.

E. Trophozoite to amoeba transformation.

*A 45-year-old man comes to the emergency department (ED) because of a 2-day history of bloody diarrhea, abdominal pain, and fever. He vomited 2 hours before coming to the ED. His temperature is 38.5°C (101.3 °F), pulse is 90/min, respirations are 15/min and blood pressure is 120/80 mm Hg. Physical examination shows generalized abdominal tenderness without guarding. Laboratory studies of his stool show Gram negative, non-lactose fermenting, motile rods. Which of the following is the most likely pathogenesis of the causal agent? A. Cerebral invasion B. Tubulin polymerization C. Protein synthesis inhibition D. Gallbladder colonization E. Vacuolar replication

E. Vacuolar replication -non typhoidal salmenelosis

A 34-year-old woman presents with recurrent episodes of severe headaches, palpitations, tachycardia, and sweating. A physical examination reveals her blood pressure to be within normal limits; however, during one of these episodes of headaches, palpitations, and tachycardia, her blood pressure is found to be markedly elevated. Workup finds a small tumor of the right adrenal gland. Which of the following is most likely to be increased in the urine of this individual? A. Acetone B. Aminolevulinic acid (ALA) C. Hydroxy-indoleacetic acid (HIAA) D. N-formiminoglutamate (FIGlu) E. Vanillylmandelic acid (VMA)

E. Vanillylmandelic acid (VMA) Medulla made from neural crest cells and treated with phenoxybezamine and phentolamine (remember to give alpha before beta!)

11. A 30-year-old HIV-positive man comes to the physician because of a 5-month history of intermittent, worsening watery diarrhea (currently 15-18 stools/day), abdominal cramping, and nausea. He has not eaten anything unusual or traveled recently, but regularly swims in a local lake. He has been non-compliant with his antiretroviral drugs for the past year. His pulse is 120/min and blood pressure is 100/70 mmHg. Physical examination shows a thin, lethargic man with diffuse abdominal tenderness. Laboratory studies show CD4 count of 85/mm3, and a thick-walled oocyst. Which of the following lab staining procedures will most likely demonstrate the presence of the causal agent? A. Giemsa stain B. Gram stain C. Methylamine silver stain D. Wright's Stain E. Ziehl-Neelsen Stain

E. Ziehl-Neelsen Stain = cryptosporidium

A 35-year-old woman comes to the physician because of a 1-day history of watery diarrhea, nausea, and vomiting. She is currently on vacation in South America. She states that she has been careful to drink only bottled water during her trip, but she tasted some local food 2 days ago from street vendors. Laboratory studies show no fecal leukocytes or RBCs. Which of the following pathogenic mechanisms is most likely responsible from her symptoms? A. Invasion and release of cytotoxin in large intestine B. Invasion and release of cytotoxin in small intestine C. Ingestion of preformed exotoxin D. colonization and release of cytotoxin in stomach E. colonization and release of enterotoxin in small intestine

E. colonization and release of enterotoxin in small intestine

A 40-year-old woman comes to the physician because of a 2-month history of progressive fatigue and cold intolerance. Her pulse is 60/min, blood pressure is 100/70mm Hg and respirations are 16/min. Physical examination shows cold, dry skin, hyporeflexia and enlargement of the thyroid gland. Thyroid function tests show increased TSH, decreased T3 and T4 levels and presence of thyroid peroxidase antibodies in the serum. This patient has an increased risk of developing which of the following complications? A. Thyrotoxicosis B. Osteoporosis C. Thyroid B-cell lymphoma D. Mental retardation E. Umbilical hernia

Primary hypothyroidism - develop AT the level of the gland, so low hormone = more TSH and TPO abx = hashimoto thyroiditis and with Sjorgens syndrome both are associated with MALToma (B-cell lymphoma) —> discussed in RHS


Conjuntos de estudio relacionados

Remediation Assignment for Exam 1

View Set

Eg101-Otto and Diesel Cycles quiz 4

View Set

Chapter 40: Management of Patients with Gastric and Duodenal Disorders

View Set

Earth Science--Weathering, Erosion, and Deposition--8th Grade

View Set

Chapter 18 and 19 Questions Review

View Set

AP Human Geography Chapter 1 Study Guide

View Set

Section 1: Origination and Underwriting

View Set